Physics-Course and Exam Description
Physics-Course and Exam Description
Course framework
Instructional
section
ample exam
S
questions
AP Physics 1
®
Effective
Fall 2021
AP Physics 1:
®
Algebra-Based
COURSE AND EXAM DESCRIPTION
Effective
Fall 2021
© 2021 College Board. College Board, Advanced Placement, AP, AP Central, and the acorn logo are
registered trademarks of College Board. All other products and services may be trademarks of their
respective owners.
COURSE FRAMEWORK
11 Introduction
13 Course Framework Components
15 Science Practices
17 Course Content
20 Course at a Glance
25 Unit Guides
25 Introduction
27 Using the Unit Guides
31 UNIT 1: Kinematics
41 UNIT 2: Dynamics
59 UNIT 3: Circular Motion and Gravitation
77 UNIT 4: Energy
91 UNIT 5: Momentum
107 UNIT 6: Simple Harmonic Motion
117 UNIT 7: Torque and Rotational Motion
LABORATORY INVESTIGATIONS
135 Lab Experiments
136 How to Set Up a Lab Program
INSTRUCTIONAL APPROACHES
141 Selecting and Using Course Materials
142 Guided Inquiry in AP Physics 1
144 Instructional Strategies
155 Developing the Science Practices
EXAM INFORMATION
169 Exam Overview
174 Sample Exam Questions
SCORING GUIDELINES
187 Question 1: Quantitative/Qualitative Translation
193 Question 2: Paragraph Argument Short Answer
APPENDIX
199 Table of Information: Equations
Acknowledgments
College Board’s Advanced Placement® Program (AP®) and students with free formative assessments—
enables willing and academically prepared students Personal Progress Checks—that teachers can assign
to pursue college-level studies—with the opportunity throughout the year to measure student progress
to earn college credit, advanced placement, or as they acquire content knowledge and develop
both—while still in high school. Through AP courses science practices.
in 38 subjects, each culminating in a challenging
exam, students learn to think critically, construct solid Enrolling Students:
arguments, and see many sides of an issue—skills
that prepare them for college and beyond. Taking Equity and Access
AP courses demonstrates to college admission officers College Board strongly encourages educators to
that students have sought the most challenging make equitable access a guiding principle for their
curriculum available to them, and research indicates AP programs by giving all willing and academically
that students who score a 3 or higher on an AP Exam prepared students the opportunity to participate
typically experience greater academic success in in AP. We encourage the elimination of barriers
college and are more likely to earn a college degree that restrict access to AP for students from ethnic,
than non-AP students. Each AP teacher’s syllabus racial, and socioeconomic groups that have been
is evaluated and approved by faculty from some of traditionally underserved. College Board also believes
the nation’s leading colleges and universities, and that all students should have access to academically
AP Exams are developed and scored by college faculty challenging coursework before they enroll in AP classes,
and experienced AP teachers. Most four-year colleges which can prepare them for AP success. It is only
and universities in the United States grant credit, through a commitment to equitable preparation and
advanced placement, or both on the basis of successful access that true equity and excellence can be achieved.
AP Exam scores; more than 3,300 institutions worldwide
annually receive AP scores. Offering AP Courses:
The AP Course Audit
AP Course Development The AP Program unequivocally supports the principle
In an ongoing effort to maintain alignment with best that each school implements its own curriculum
practices in college-level learning, AP courses and that will enable students to develop the content
exams emphasize challenging, research-based understandings and science practices described in the
curricula aligned with higher education expectations. course framework.
Individual teachers are responsible for designing their While the unit sequence represented in this publication
own curriculum for AP courses, selecting appropriate is optional, the AP Program does have a short list of
college-level readings, assignments, and resources. curricular and resource requirements that must be
This course and exam description presents the fulfilled before a school can label a course “Advanced
content and science practices that are the focus of Placement” or “AP.” Schools wishing to offer AP courses
the corresponding college course and that appear must participate in the AP Course Audit, a process
on the AP Exam. It also organizes the content and through which AP teachers’ course materials are
science practices into a series of units that represent reviewed by college faculty. The AP Course Audit
a sequence found in widely adopted college textbooks was created to provide teachers and administrators
and that many AP teachers have told us they follow in with clear guidelines on curricular and resource
order to focus their instruction. The intention of this requirements for AP courses and to help colleges and
publication is to respect teachers’ time and expertise universities validate courses marked “AP” on students’
by providing a roadmap that they can modify and adapt transcripts. This process ensures that AP teachers’
to their local priorities and preferences. Moreover, by courses meet or exceed the curricular and resource
organizing the AP course content and science practices expectations that college and secondary school faculty
into units, the AP Program is able to provide teachers have established for college-level courses.
Committee members are selected to represent a variety Using and Interpreting AP Scores
of perspectives and institutions (public and private,
The extensive work done by college faculty and
small and large schools and colleges) and a range of
AP teachers in the development of the course and
gender, racial/ethnic and regional groups. A list of each
exam and throughout the scoring process ensures
subject’s current AP Test Development Committee
that AP Exam scores accurately represent students’
members is available on apcentral.collegeboard.org.
achievement in the equivalent college course. Frequent
Throughout AP course and exam development, and regular research studies establish the validity of
College Board gathers feedback from various AP scores as follows:
stakeholders in both secondary schools and higher
education institutions. This feedback is carefully Credit College Grade
considered to ensure that AP courses and exams are AP Score Recommendation Equivalent
able to provide students with a college-level learning
5 Extremely well qualified A
experience and the opportunity to demonstrate their
qualifications for advanced placement or college credit. 4 Well qualified A-, B+, B
The exam scoring process, like the course and exam Possibly qualified n/a
2
development process, relies on the expertise of both
AP teachers and college faculty. While multiple-choice 1 No recommendation n/a
questions are scored by machine, the free-response
By completing a simple activation process at the start of the school year, teachers and
students receive access to a robust set of classroom resources.
AP Classroom
AP Classroom is a dedicated online platform designed to support teachers and students
throughout their AP experience. The platform provides a variety of powerful resources and
tools to provide yearlong support to teachers and enable students to receive meaningful
feedback on their progress.
UNIT GUIDES
Appearing in this publication and on AP Classroom, these planning guides outline all required
course content and science practices, organized into commonly taught units. Each unit guide
suggests a sequence and pacing of content, scaffolds skill instruction across units, organizes
content into topics, and provides tips on taking the AP Exam.
PROGRESS DASHBOARD
This dashboard allows teachers to review class and individual student progress throughout
the year. Teachers can view class trends and see where students struggle with content and
science practices that will be assessed on the AP Exam. Students can view their own progress
over time to improve their performance before the AP Exam.
AP QUESTION BANK
This online library of real AP Exam questions provides teachers with secure questions to
use in their classrooms. Teachers can find questions indexed by course topics and science
practices, create customized tests, and assign them online or on paper. These tests enable
students to practice and get feedback on each question.
§§ Teachers and students sign in to, or create, their College Board accounts.
§§ Teachers confirm that they have added the course they teach to their AP Course Audit
account and have had it approved by their school’s administrator.
§§ Teachers or AP Coordinators, depending on who the school has decided is responsible,
set up class sections so students can access AP resources and have exams ordered on
their behalf.
§§ Students join class sections with a join code provided by their teacher or AP coordinator.
§§ Students will be asked for additional registration information upon joining their first class
section, which eliminates the need for extensive answer sheet bubbling on exam day.
While the digital activation process takes a short time for teachers, students, and
AP coordinators to complete, overall it helps save time and provides the following
additional benefits:
Integrating AP resources throughout the course can help students develop the course
science practices and conceptual understandings. The instructional model outlined below
shows possible ways to incorporate AP resources into the classroom.
Plan
Teachers may consider the following approaches as they plan their instruction before
teaching each unit.
§§ Review the overview at the start of each unit guide to identify essential questions,
conceptual understandings, and science practices for each unit.
§§ Use the Unit at a Glance table to identify related topics that build toward a common
understanding, and then plan appropriate pacing for students.
§§ Identify useful strategies in the Instructional Approaches section to help teach the
concepts and science practices.
Teach
When teaching, supporting resources can be used to build students’ conceptual
understanding and mastery of science practices.
§§ Use the topic pages in the unit guides to identify the required content.
§§ Integrate the content with a skill, considering any appropriate scaffolding.
§§ Employ any of the instructional strategies previously identified.
§§ Use the available resources on the topic pages to bring a variety of assets into
the classroom.
Assess
Teachers can measure student understanding of the content and science practices covered
in the unit and provide actionable feedback to students.
§§ At the end of each unit, use AP Classroom to assign students the online Personal Progress
Checks, as homework or as an in-class task.
§§ Provide question-level feedback to students through answer rationales; provide unit- and
skill-level feedback using the progress dashboard.
§§ Create additional practice opportunities using the AP Question Bank and assign them
through AP Classroom.
Prerequisites
There are no prerequisite courses. Students should have completed Geometry and be
concurrently taking Algebra II or an equivalent course. Although the Physics 1 course
includes basic use of trigonometric functions, this understanding can be gained either in the
concurrent math course or in the AP Physics 1 course itself.
Laboratory Requirement
This course requires that twenty-five percent of instructional time will be spent in hands-on
laboratory work, with an emphasis on inquiry-based investigations that provide students
with opportunities to demonstrate the foundational physics principles and apply the
science practices.
Colleges may require students to present their laboratory materials from AP science courses
before granting college credit for laboratory work, so students should be encouraged to retain
their laboratory notebooks, reports, and other materials.
Course
Framework
Introduction
This document is not a complete curriculum. Teachers create their own local curriculum by selecting, for each
concept, content that enables students to explore the course learning objectives and meets state or local
requirements. The result is a course that prepares students for college credit and placement.
Overview
This course framework provides a clear and detailed description of the course
requirements necessary for student success. The framework specifies what
students must know, be able to do, and understand to qualify for college credit
or placement.
2 COURSE CONTENT
The course content is organized into commonly taught units of study that
provide a suggested sequence for the course and detail required content
and conceptual understandings that colleges and universities typically
expect students to master to qualify for college credit and/or placement.
This content is grounded in big ideas, which are cross-cutting concepts
that build conceptual understanding and spiral throughout the course.
Science
Practices
The table that follows presents the science practices that students should develop
during the AP Physics 1 course. These practices form the basis of many tasks on
the AP Physics 1 Exam.
The unit guides that follow embed and spiral these practices throughout the
course, providing teachers with one way to integrate the practices into the course
content with sufficient repetition to prepare students to transfer those science
practices when taking the AP Physics 1 Exam.
More detailed information about teaching the science practices can be found in the
Instructional Approaches section of this publication.
Science Practices
Practice 1 Practice 2 Practice 3 Practice 4 Practice 5 Practice 6 Practice 7
1.1 The student can 2.1 The student can 3.1 The student 4.1 The student can 5.1 The student can 6.1 The student can 7.1 The student can
Course
Content
Based on the Understanding by Design® (Wiggins and McTighe) model, this course
framework provides a clear and detailed description of the course requirements
necessary for student success. The framework specifies what students must know,
be able to do, and understand, with a focus on six big ideas that encompass core
principles, theories, and processes of physics. The framework also encourages
instruction that prepares students to make connections across domains through
a broader way of thinking about the physical world.
Big Ideas
The big ideas serve as the foundation of the course and allow students to create
meaningful connections among concepts. They are often abstract concepts or themes
that become threads that run throughout the course. Revisiting the big ideas and
applying them in a variety of contexts allows students to develop deeper conceptual
understanding. Below are the big ideas of the course and a brief description of each.
1-Systems
SYS
2-Fields
3-Force
Interactions
INT
4-Change
CHA
5-Conservation
CON
+ and Acceleration 1
7
The Course at a Glance provides
a useful visual organization FLD 2.2 The Gravitational
CHA 1.2 Representations of
of the AP Physics 1 course 2 Field
components, including: + Motion
7
PRACTICES
Science practices spiral throughout the course.
1 Modeling 4 Experimental
Methods
2 Mathematical
Routines 5 Data Analysis
3 Scientific 6 Argumentation
Questioning
7 Making
Connections
INT 3-Force
Interactions
Assess
Assign the Personal
Progress Checks—either as
homework or in class—for Personal Progress Check 1 Personal Progress Check 2
each unit. Each Personal Multiple-choice: ~15 questions Multiple-choice: ~40 questions
Progress Check contains Free-response: 2 questions Free-response: 2 questions
formative multiple-choice and § Experimental Design § Quantitative/Qualitative Translation
§ Paragraph Argument Short Answer § Short Answer
free-response questions. The
feedback from these checks
shows students the areas where
they need to focus.
V.1 | 20
© 2021 College Board
UNIT Circular Motion UNIT UNIT
Energy Momentum
3 and Gravitation 4 5
~8-10 Class
Periods 6-8 % AP Exam
Weighting ~22-25 Class
Periods 20-28% AP Exam
Weighting ~14-17 Class
Periods 12-18% AP Exam
Weighting
FLD 3.1 Vector Fields CON 4.1 Open and Closed INT 5.1 Momentum and
Systems: Energy
+ Impulse
6
7
INT 3.2 Fundamental Forces
7 INT 4.2 Work and Mechanical CHA 5.2 Representations of
Energy
+ Changes in Momentum
CHA
INT
2
3.3 Gravitational and
Electric Forces
+
7 CON 5.3 Open and Closed
CON 4.3 Conservation of 6 Systems: Momentum
7
Gravitational Field/
+
FLD 3.4 Energy, the Work-
2 Acceleration Due Energy Principle,
7
to Gravity on and Power
CON 5.4 Conservation of
Different Planets + Linear Momentum
V.1 | 21
© 2021 College Board
UNIT Simple Harmonic UNIT Torque and
6 Motion 7 Rotational Motion
~4-7 Class
Periods 4-6% AP Exam
Weighting ~14-19 Class
Periods 12-18% AP Exam
Weighting
V.1 | 22
© 2021 College Board
THIS PAGE IS INTENTIONALLY LEFT BLANK.
THIS PAGE IS INTENTIONALLY LEFT BLANK.
AP PHYSICS 1
Unit
Guides
Introduction
Designed with input from the community of AP Physics 1 educators, the
unit guides offer teachers helpful guidance in building students’ skills and
knowledge. The suggested sequence was identified through a thorough
analysis of the syllabi of highly effective AP teachers and the organization of
typical college textbooks.
UNIT OPENERS
UNIT The Unit Overview contextualizes and situates the key content of
1 12–18 ~19–22 CLASS PERIODS
%
AP EXAM WEIGHTING
the unit within the scope of the course. It also describes specific
aspects of the science practices that are appropriate to focus on
Kinematics in that unit.
The science practices for each topic link the content in that
Understanding
Class Periods
Enduring
1.2 Representations 1.2 The student can describe representations and models of
of Motion natural or man-made phenomena and systems in the domain.*
*Indicates a science practice not assessed with its paired topic on this unit’s Personal Progress Check.
TOPIC PAGES
UNIT Enduring understandings are the long-term takeaways related
Energy
4
to the big ideas that leave a lasting impression on students.
TOPIC 4.3 Students build and earn these understandings over time by
SCIENCE PRACTICES
Modeling
Mathematical
Routines
ENDURING UNDERSTANDING
2.1
The student can justify the
designed to help teachers integrate science practices [SP] with
specific content and to provide them with clear information on how
5.B selection of a mathematical
The energy of a system is conserved. routine to solve problems.
2.2
how the same big ideas and enduring understandings are applied
enduring understandings
and/or big ideas.
Note: Labels are used to distinguish each unique element of the required course content and are used throughout this course and exam
description. Additionally, they are used in the AP Question Bank and other resources found in AP Classroom. Big ideas are labeled by number,
with “1” referring to SYS, “2” referring to FLD, “3” referring to INT, “4” referring to CHA, “5” referring to CON, and “6” referring to WAV. The Course
at a Glance on p. 20 contains more information on how the big ideas are numbered. Enduring understandings are labeled sequentially according
to the big idea that they are related to. Essential knowledge statements are labeled to correspond with the enduring understanding they
relate to. Finally, learning objectives are labeled to correspond with the essential knowledge statement they relate to.
UNIT 1
Kinematics
12–18%
AP EXAM WEIGHTING
~19–22
CLASS PERIODS
Kinematics
Unit Overview
BIG IDEA 3 The world is in a constant state of motion. To understand the world, students must first
Force Interactions INT understand movement. Unit 1 introduces students to the study of motion and serves as a
§ How can the motion of foundation for all of AP Physics 1 by beginning to explore the complex idea of acceleration and
objects be predicted showing them how representations can be used to model and analyze scientific information
and/or explained? as it relates to the motion of objects. By studying kinematics, students will learn to represent
§ Can equations be used
motion—both uniform and accelerating—in narrative, graphical, and/or mathematical forms
to answer questions
and from different frames of reference. These representations will help students analyze the
regardless of the specific motion of objects and systems while also dispelling some common misconceptions
questions’ specificity? they may have about motion, such as exclusively using negative acceleration to describe
an object slowing down. Additionally, students will have the opportunity to go beyond their
§ How can the idea of
traditional understanding of mathematics. Instead of solving equations, students will use them
frames of reference
to support their reasoning and tighten their grasp on the laws of physics. Lastly, students will
allow two people to
begin making predictions about motion and justifying claims with evidence by exploring the
tell the truth yet have
relationships between the physical quantities of acceleration, velocity, position, and time. This
conflicting reports?
is an important starting point for students, as these fundamental science practices will spiral
throughout the course and appear in multiple units.
BIG IDEA 4
Change CHA
§ How can we use Preparing for the AP Exam
models to help us
On the AP Physics 1 Exam, there is an experimental design question in the free-response
understand motion?
section that is worth 12 points. Students must be able to justify their selection of the kind of
§ Why is the general rule data needed to answer the question and then design a plan to collect that data.
for stopping your car
“when you double your When presented with an experimental design question, students often do not know where to
speed, you must give start. Students should be given scaffolded opportunities to determine the appropriate data
yourself four times as needed to answer a scientific question. To create laboratory experiments for students who
much distance to stop?” struggle with identifying the data needed to answer a particular question, please refer to the
learning objectives linked to this unit.
1 Kinematics
UNIT AT A GLANCE
Understanding
Class Periods
Enduring
Position, Velocity,
1.1 1.5 The student can re-express key elements of natural
and Acceleration phenomena across multiple representations in the domain.
Representations
1.2 1.2 The student can describe representations and models of
of Motion natural or man-made phenomena and systems in the domain.*
*Indicates a science practice not assessed with its paired topic on this unit’s Personal Progress Check.
1 Kinematics
SCIENCE PRACTICES
Modeling
TOPIC 1.1
1.5
The student can re-
express key elements of
Position, Velocity,
natural phenomena across
multiple representations in
the domain.
and Acceleration
Mathematical
Routines
2.1
The student can justify the
selection of a mathematical Required Course Content
routine to solve problems.
2.2
The student can apply
mathematical routines to
ENDURING UNDERSTANDING
quantities that describe
natural phenomena. 3.A
All forces share certain common characteristics when considered by observers in
Experimental inertial reference frames.
Method
4.2
The student can design
LEARNING OBJECTIVE ESSENTIAL KNOWLEDGE
a plan for collecting data
to answer a particular 3.A.1.1 3.A.1
scientific question. Express the motion of an An observer in a reference frame can describe
object using narrative, the motion of an object using such quantities
Data Analysis
mathematical, and as position, displacement, distance, velocity,
5.1 graphical representations. speed, and acceleration.
The student can analyze [SP 1.5, 2.1, 2.2] a. Displacement, velocity, and acceleration are
data to identify patterns
3.A.1.2 all vector quantities.
or relationships.
Design an experimental b. Displacement is change in position. Velocity
investigation of the motion of is the rate of change of position with
an object. [SP 4.2] time. Acceleration is the rate of change of
3.A.1.3 velocity with time. Changes in each property
Analyze experimental data are expressed by subtracting initial values
describing the motion of from final values.
an object and be able to Relevant Equations:
express the results of the Δx
analysis using narrative, v avg =
mathematical, and graphical
Δt
representations. [SP 5.1]
Δv
aavg =
Δt
c. A choice of reference frame determines
the direction and the magnitude of each of
these quantities.
d. There are three fundamental interactions
or forces in nature: the gravitational force,
the electroweak force, and the strong force.
The fundamental forces determine both the
structure of objects and the motion of objects.
BOUNDARY STATEMENT:
AP Physics 2 has learning objectives under
Enduring Understanding 3.A that focus
on electric and magnetic forces and other
forces arising in the context of interactions
introduced in Physics 2, rather than the
mechanical systems introduced in Physics 1.
1 Kinematics
SCIENCE PRACTICES
Modeling
TOPIC 1.2
1.2
The student can describe
representations and models
Representations
of natural or man-made
phenomena and systems in
the domain.
of Motion
1.4
The student can use
representations and models
to analyze situations or
solve problems qualitatively
and quantitatively. Required Course Content
Mathematical
Routines
2.2
ENDURING UNDERSTANDING
The student can apply
mathematical routines to 4.A
quantities that describe The acceleration of the center
natural phenomena.
of mass of a system is related to the net force exerted
F
2.3 on the system, where a = .
m
The student can estimate
quantities that describe
natural phenomena. LEARNING OBJECTIVE ESSENTIAL KNOWLEDGE
Argumentation 4.A.1.1 4.A.1
UNIT 2
Dynamics
16–20%
AP EXAM WEIGHTING
~21–24
CLASS PERIODS
Dynamics
Unit Overview
BIG IDEA 1 In Unit 2, students are introduced to the term force, which is the interaction of an object with
Systems SYS another object. Part of the larger study of dynamics, forces are used as the lens through
§ How can the properties of which students analyze and come to understand a variety of physical phenomena. This
internal and gravitational is accomplished by revisiting and building upon the representations presented in Unit 1,
mass be experimentally specifically the introduction to the free-body diagram. Translation, however, is key in this
verified to be the same? unit: Students must be able to portray the same object–force interactions through different
§ How do you decide
graphs, diagrams, and mathematical relationships. Students will continue to make meaning
what to believe about from models and representations that will help them further analyze systems, the interactions
scientific claims? between systems, and how these interactions result in change.
§ How does something we Alongside mastering the use of specific force equations, Unit 2 also encourages students
cannot see determine to derive new expressions from fundamental principles to help them make predictions in
how an object behaves? unfamiliar, applied contexts. The skill of making predictions will be nurtured throughout the
course to help students craft sound scientific arguments.
BIG IDEA 2
Fields FLD
§ How do objects
Preparing for the AP Exam
with mass respond The AP Physics 1 Exam requires students to be able to re-express key elements of natural
when placed in a phenomena across multiple representations in the domain. This skill appears in the
gravitational field? Qualitative/Quantitative Translation (QQT), a long free-response question that requires
§ Why is the acceleration students to go between words and mathematics in describing and analyzing a situation.
due to gravity constant A QQT question might ask students to work with multiple representations or to evaluate another
on Earth’s surface? student’s words or representations. Representations include mathematical equations,
narrative descriptions, graphs, diagrams, and data tables.
BIG IDEA 3
Students who have primarily been exposed to numerical problem solving often struggle with
Force Interactions INT
a QQT question because it requires students to have a more conceptual understanding
§ Are different kinds of
of both content and representations. Opportunities to translate between different
forces really different?
representations, including equations, diagrams, graphs, and written descriptions, will help
§ How can Newton’s
students prepare for the QQT question.
laws of motion be
used to predict the
behavior of objects?
BIG IDEA 4
Change CHA
§ Why does the same
push change the motion
of a shopping cart more
than the motion of a car?
2 Dynamics
UNIT AT A GLANCE
Understanding
Class Periods
Enduring
Systems
2.1 1.1 The student can create representations and models of
natural or man-made phenomena and systems in the domain.*
1.A
The Gravitational
2.2 2.2 The student can apply mathematical routines to
Field quantities that describe natural phenomena.
2.B
Contact Forces
2.3 6.1 The student can justify claims with evidence.
3.C
Newton’s Third
2.5 1.1 The student can create representations and models of
Law and Free-Body natural or man-made phenomena and systems in the domain.
Diagrams
1.4 The student can use representations and models
to analyze situations or solve problems qualitatively
and quantitatively.
*Indicates a science practice not assessed with its paired topic on this unit’s Personal Progress Check.
Class Periods
Enduring
Newton’s
2.6 1.1 The student can create representations and models of
Second Law natural or man-made phenomena and systems in the domain.
*Indicates a science practice not assessed with its paired topic on this unit’s Personal Progress Check.
2 Dynamics
Class Periods
Enduring
Applications
2.7 1.2 The student can describe representations and models of
of Newton’s natural or man-made phenomena and systems in the domain.*
Second Law
1.4 The student can use representations and models
to analyze situations or solve problems qualitatively
and quantitatively.
*Indicates a science practice not assessed with its paired topic on this unit’s Personal Progress Check.
5 2.7 Troubleshooting
Students take some force-related problem from the homework or textbook (one that
requires setting up Newton’s second law and maybe more). Students write out a detailed
solution that has exactly one mistake in it (not a calculation error). Post everyone’s
problems/ solutions, and then ask students to identify everyone else’s errors. The last
student to have his or her error found wins.
2 Dynamics
SCIENCE PRACTICES
Modeling
TOPIC 2.1
1.1
The student can create
representations and models
Systems
of natural or man-made
phenomena and systems in
the domain.
Making Connections
7.1
The student can connect
phenomena and models
across spatial and Required Course Content
temporal scales.
ENDURING UNDERSTANDING
1.A
The internal structure of a system determines many properties of the system.
1.A.5.1 1.A.5
Model verbally or visually Systems have properties that are determined
the properties of a system by the properties and interactions of
based on its substructure their constituent atomic and molecular
and relate this to changes in substructures. In AP Physics, when the
the system properties over properties of the constituent parts are not
time as external variables are important in modeling the behavior of the
changed. [SP 1.1, 7.1] macroscopic system, the system itself may be
referred to as an object.
Making Connections
7.2
The student can connect
concepts in and across
Required Course Content domain(s) to generalize or
extrapolate in and/or across
enduring understandings
and/or big ideas.
ENDURING UNDERSTANDING
2.B
A gravitational field is caused by an object with mass.
2 Dynamics
SCIENCE PRACTICES
Argumentation
TOPIC 2.3
6.1
The student can justify
claims with evidence.
Contact Forces
6.2
The student can
construct explanations
of phenomena based on
evidence produced through
scientific practices.
ENDURING UNDERSTANDING
3.C
At the macroscopic level, forces can be categorized as either long-range (action-at-
a-distance) forces or contact forces.
ENDURING UNDERSTANDING
1.C
Objects and systems have properties of inertial mass and gravitational mass that are
experimentally verified to be the same and that satisfy conservation principles.
1.C.3.1 1.C.3
Design a plan for collecting Objects and systems have properties of
data to measure gravitational inertial mass and gravitational mass that are
mass and inertial mass and to experimentally verified to be the same and that
distinguish between the two satisfy conservation principles.
experiments. [SP 4.2]
2 Dynamics
SCIENCE PRACTICES
Modeling
TOPIC 2.5
1.1
The student can create
representations and models
Newton’s Third Law
of natural or man-made
phenomena and systems in
the domain.
and Free-Body
1.4
The student can use
representations and models
Diagrams
to analyze situations or
solve problems qualitatively
and quantitatively. Required Course Content
Argumentation
6.1
The student can justify
ENDURING UNDERSTANDING
claims with evidence.
3.A
6.2
The student can All forces share certain common characteristics when considered by observers in
construct explanations inertial reference frames.
of phenomena based on
evidence produced through
scientific practices.
LEARNING OBJECTIVE ESSENTIAL KNOWLEDGE
6.4
3.A.2.1 3.A.2
The student can make
claims and predictions Represent forces in diagrams Forces are described by vectors.
about natural phenomena or mathematically using a. Forces are detected by their influence on
based on scientific theories appropriately labeled vectors the motion of an object.
and models. with magnitude, direction,
b. Forces have magnitude and direction.
and units during the analysis
Making Connections of a situation. [SP 1.1]
7.2
The student can connect
concepts in and across 3.A.3.1 3.A.3
domain(s) to generalize or
Analyze a scenario and make A force exerted on an object is always
extrapolate in and/or across
claims (develop arguments, due to the interaction of that object with
enduring understandings
and/or big ideas.
justify assertions) about another object.
the forces exerted on an a. An object cannot exert a force on itself.
object by other objects for
b. Even though an object is at rest, there
different types of forces
may be forces exerted on that object
or components of forces.
[SP 6.4, 7.2] by other objects.
c. The acceleration of an object, but not
3.A.3.2
necessarily its velocity, is always in the
Challenge a claim that an direction of the net force exerted on the
object can exert a force on object by other objects.
itself. [SP 6.1]
3.A.3.3
Describe a force as an
interaction between two
objects, and identify both
objects for any force. [SP 1.4]
2 Dynamics
SCIENCE PRACTICES
Modeling
TOPIC 2.6
1.1
The student can create
representations and models
Newton’s Second Law
of natural or man-made
phenomena and systems in
the domain.
1.4
The student can use
representations and models
to analyze situations or
solve problems qualitatively
and quantitatively. Required Course Content
1.5
The student can re-
express key elements of
natural phenomena across
ENDURING UNDERSTANDING
multiple representations in
the domain. 3.B
Classically, the acceleration
of an object interacting with other objects can be
Mathematical
Routines
F
predicted by using a = .
2.2
m
The student can apply
mathematical routines to LEARNING OBJECTIVE ESSENTIAL KNOWLEDGE
quantities that describe
natural phenomena. 3.B.1.1 3.B.1
Predict the motion of an If an object of interest interacts with several
Experimental object subject to forces other objects, the net force is the vector sum
Method exerted by several objects of the individual forces. Projectile motion
4.2 using an application of and circular motion are both included in
The student can design Newton’s second law in a AP Physics 1.
a plan for collecting data variety of physical situations, Relevant Equation:
to answer a particular with acceleration in one
scientific question.
F F net
dimension. [SP 6.4, 7.2] a= =
3.B.1.2
m m
Data Analysis
5.1
Design a plan to collect and
BOUNDARY STATEMENT:
analyze data for motion (static,
The student can analyze AP Physics 2 contains learning objectives
data to identify patterns
constant, or accelerating) from
force measurement, and carry for Enduring Understanding 3.B that focus
or relationships.
out an analysis to determine on electric and magnetic forces and other
the relationship between forces arising in the context of interactions
the net force and the vector introduced in Physics 2, rather than the
sum of the individual forces. mechanical systems introduced in Physics 1.
[SP 4.2, 5.1]
3.B.1.3
Re-express a free-body
diagram into a mathematical
representation, and solve the
mathematical representation
for the acceleration of the
object. [SP 1.5, 2.2]
2 Dynamics
SCIENCE PRACTICES
Modeling
TOPIC 2.7
1.2
The student can describe
representations and models
Applications of
of natural or man-made
phenomena and systems in
the domain.
Newton’s Second Law
1.4
The student can use
representations and models
to analyze situations or
solve problems qualitatively
and quantitatively. Required Course Content
Mathematical
Routines
2.2
ENDURING UNDERSTANDING
The student can apply
mathematical routines to 4.A
quantities that describe The acceleration of the center
of mass of a system is related to the net force exerted
natural phenomena. F
2.3 on the system, where a = .
m
The student can estimate
quantities that describe
natural phenomena. LEARNING OBJECTIVE ESSENTIAL KNOWLEDGE
Data Analysis 4.A.1.1 4.A.1
4.A.3.1 4.A.3
Apply Newton’s second law Forces that the systems exert on each other
to systems to calculate the are due to interactions between objects in the
change in the center-of-mass systems. If the interacting objects are parts of
velocity when an external the same system, there will be no change in the
force is exerted on the center-of-mass velocity of that system.
system. [SP 2.2] Relevant Equation:
4.A.3.2
ΣF F net
Use visual or mathematical a= =
representations of the forces
msystem m
between objects in a system
to predict whether or not
there will be a change in the
center-of-mass velocity of
that system. [SP 1.4]
UNIT 3
Circular
Motion and
Gravitation
6–8%
AP EXAM WEIGHTING
~8–10
CLASS PERIODS
Unit Overview
BIG IDEA 1 In Unit 3, students will continue to enhance their understanding of the physical world using
Systems SYS models and representations to create a more complete and complex model of motion,
§ How does changing the particularly as it relates to gravitational mass and inertial mass. Again, translation and
mass of an object affect connections are essential—students must be able to use content and science practices from
the gravitational force? the previous two units and apply them in different ways.
§ Why is a refrigerator While it’s essential that students are able to calculate numerical answers to questions,
hard to push in space? it is more important that they can combine mathematical representations to make new
representations that more accurately describe natural phenomena. For example, students
BIG IDEA 2 should be comfortable combining equations for uniform circular motion with gravitational
Fields FLD
equations to describe the circular path of a satellite circling a planet.
§ Why do we feel pulled
toward Earth but not It is also vital that students are given opportunities to think about and discuss the impact that
toward a pencil? changes or modifications have on physical scenarios. For example, students should be able
§ How can the to use mathematical and graphical representations to determine how doubling the distance
acceleration due to of a satellite from a planet will change the period of orbit and then justify their answer with
gravity be modified? evidence and reasoning. Specific preconceptions will be addressed in this unit, such as the
idea of a centrifugal force. Students will also have opportunities to wrestle with the idea of
BIG IDEA 3 field models, which will be expanded upon in Unit 8.
Force Interactions INT
§ How can Newton’s Preparing for the AP Exam
laws of motion be used
to predict the behavior Students will be asked to give a paragraph-length response to demonstrate their ability to
of objects? communicate their understanding of a physical situation in a reasoned, expository analysis.
For full credit, the response should be a coherent, organized, and sequential description of the
§ How can we use forces
analysis of a situation that draws from evidence, cites physical principles, and clearly presents
to predict the behavior of
the student’s thinking. Full credit may not be earned if the response contains any of the
objects and keep us safe?
following: principles not presented in a logical order, lengthy digressions within an argument,
BIG IDEA 4 or a lack of linking prose between equations or diagrams.
Change CHA
Students will also be asked to explain phenomena based on evidence produced through
§ How is the acceleration
scientific practices while using mathematical routines as evidence for claims. Students who
of the center of mass of
are weak mathematically will need significant scaffolding to help them develop the conceptual
a system related to the
mathematical understanding necessary to succeed on the AP Physics 1 Exam.
net force exerted on the
system?
§ Why is it more difficult
to stop a fully loaded
dump truck than a small
passenger car?
UNIT AT A GLANCE
Understanding
Class Periods
Enduring
Fundamental Forces
3.2 7.1 The student can connect phenomena and models across
3.G
Gravitational and
3.3 2.2 The student can apply mathematical routines to
Electric Forces quantities that describe natural phenomena.
3.C
Gravitational
3.4 2.2 The student can apply mathematical routines to
Field/Acceleration quantities that describe natural phenomena.
Due to Gravity on
2.B
Inertial vs.
3.5 4.2 The student can design a plan for collecting data to
1.C
Centripetal
3.6 5.3 The students can evaluate the evidence provided by data
sets in relation to a particular scientific question.
4.A
Acceleration and
Centripetal Force
Free-Body
3.7 1.1 The student can create representations and models of
Diagrams for natural or man-made phenomena and systems in the domain.
Objects in Uniform 1.4 The student can use representations and models
Circular Motion to analyze situations or solve problems qualitatively
and quantitatively.
1.5 The student can re-express key elements of natural
phenomena across multiple representations in the domain.
3.B
*Indicates a science practice not assessed with its paired topic on this unit’s Personal Progress Check.
Class Periods
Enduring
Applications of
3.8 1.1 The student can create representations and models of
Circular Motion natural or man-made phenomena and systems in the domain.
and Gravitation 1.4 The student can use representations and models
to analyze situations or solve problems qualitatively
and quantitatively.
1.5 The student can re-express key elements of natural
phenomena across multiple representations in the domain.
2.1 The student can justify the selection of a mathematical
routine to solve problems.
2.2 The student can apply mathematical routines to
quantities that describe natural phenomena.
3.A
ENDURING UNDERSTANDING
2.A
A field associates a value of some physical quantity with every point in space. Field
models are useful for describing interactions that occur at a distance (long-range
forces), as well as a variety of other physical phenomena.
BOUNDARY STATEMENT:
Physics 1 treats gravitational fields; Physics 2
treats electric and magnetic fields.
SCIENCE PRACTICE
Making Connections
TOPIC 3.2
7.1
The student can connect
phenomena and models
Fundamental Forces
across spatial and
temporal scales.
ENDURING UNDERSTANDING
3.G
Certain types of forces are considered fundamental.
Gravitational and
Routines
2.2
The student can apply
Making Connections
7.2
The student can connect
concepts in and across
Required Course Content domain(s) to generalize or
extrapolate in and/or across
enduring understandings
and/or big ideas.
ENDURING UNDERSTANDING
3.C
At the macroscopic level, forces can be categorized as either long-range (action-at-
a-distance) forces or contact forces.
SCIENCE PRACTICES
Mathematical
TOPIC 3.4
Gravitational
Routines
2.2
The student can apply
mathematical routines to
quantities that describe
natural phenomena.
Field/Acceleration
Making Connections Due to Gravity on
Different Planets
7.2
The student can connect
concepts in and across
domain(s) to generalize or
extrapolate in and/or across
enduring understandings
and/or big ideas. Required Course Content
ENDURING UNDERSTANDING
2.B
A gravitational field is caused by an object with mass.
SCIENCE PRACTICE
Experimental
TOPIC 3.5
Inertial vs.
Method
4.2
The student can design
a plan for collecting data
to answer a particular
scientific question.
Gravitational Mass
ENDURING UNDERSTANDING
1.C
Objects and systems have properties of inertial mass and gravitational mass that are
experimentally verified to be the same and that satisfy conservation principles.
1.C.3.1 1.C.3
Design a plan for collecting Objects and systems have properties of
data to measure gravitational inertial mass and gravitational mass that are
mass and to measure experimentally verified to be the same and that
inertial mass and to satisfy conservation principles.
distinguish between the two
experiments. [SP 4.2]
Centripetal 5.3
The student can
evaluate the evidence
Centripetal Force
Required Course Content
ENDURING UNDERSTANDING
4.A
The acceleration of the center of mass of a system
F
is related to the net force exerted on the system, where a = .
m
SCIENCE PRACTICES
Modeling
TOPIC 3.7
1.1
The student can create
representations and models
Free-Body Diagrams
of natural or man-made
phenomena and systems in
the domain.
for Objects in Uniform
1.4
The student can use
representations and models
Circular Motion
to analyze situations or
solve problems qualitatively
and quantitatively. Required Course Content
1.5
The student can re-
express key elements of
natural phenomena across
ENDURING UNDERSTANDING
multiple representations
in the domain. 3.B
Classically, the acceleration of an object ΣF
Mathematical interacting with other objects can be predicted by using a = .
Routines m
2.2
The student can apply
mathematical routines to
LEARNING OBJECTIVE ESSENTIAL KNOWLEDGE
quantities that describe 3.B.1.2 3.B.1
natural phenomena. Design a plan to collect If an object of interest interacts with several
and analyze data for other objects, the net force is the vector sum
Experimental
motion (static, constant, of the individual forces. Projectile motion
Method
or accelerating) from force and circular motion are both included in
4.2 measurements, and carry AP Physics 1.
The student can design out an analysis to determine Relevant Equation:
a plan for collecting data the relationship between
to answer a particular F F net
the net force and the vector a= =
scientific question.
sum of the individual forces. m m
[SP 4.2, 5.1]
Data Analysis
BOUNDARY STATEMENT:
3.B.1.3
5.1
Re-express a free-body AP Physics 2 contains learning objectives
The student can analyze
diagram representation into a for Enduring Understanding 3.B that focus
data to identify patterns
or relationships. mathematical representation, on electric and magnetic forces and other
and solve the mathematical forces arising in the context of interactions
representation for the introduced in Physics 2, rather than the
acceleration of the object. mechanical systems introduced in Physics 1.
[SP1.5, 2.2]
SCIENCE PRACTICES
Modeling
TOPIC 3.8
1.1
The student can create
representations and models
Applications of
of natural or man-made
phenomena and systems in
the domain.
Circular Motion
1.4
The student can use
representations and models
and Gravitation
to analyze situations or
solve problems qualitatively
and quantitatively. Required Course Content
1.5
The student can re-
express key elements of
natural phenomena across
ENDURING UNDERSTANDING
multiple representations
in the domain. 3.A
All forces share certain common characteristics when considered by observers in
Mathematical inertial reference frames.
Routines
2.1
The student can justify the
selection of a mathematical
LEARNING OBJECTIVE ESSENTIAL KNOWLEDGE
routine to solve problems. 3.A.1.1 3.A.1
2.2 Express the motion of an An observer in a reference frame can describe
The student can apply object using narrative, the motion of an object using such quantities
mathematical routines to mathematical, and as position, displacement, distance, velocity,
quantities that describe graphical representations. speed, and acceleration.
natural phenomena. [SP 1.5, 2.1, 2.2] a. Displacement, velocity, and acceleration are
Experimental 3.A.1.2 all vector quantities.
Method Design an experimental b. Displacement is change in position. Velocity
4.2
investigation of the motion of is the rate of change of position with
an object. [SP 4.2] time. Acceleration is the rate of change of
The student can design
a plan for collecting data 3.A.1.3 velocity with time. Changes in each property
to answer a particular Analyze experimental are expressed by subtracting initial values
scientific question. data describing the from final values.
motion of an object and Relevant Equations:
Data Analysis
express the results of the Δx
5.1 analysis using narrative, v avg =
The student can analyze mathematical, and graphical
Δt
data to identify patterns representations. [SP 5.1]
Δv
or relationships. aavg =
Δt
c. A choice of reference frame determines
the direction and the magnitude of each of
these quantities.
d. There are three fundamental interactions
or forces in nature: the gravitational force,
the electroweak force, and the strong force.
The fundamental forces determine both the
structure of objects and the motion of objects.
BOUNDARY STATEMENT:
AP Physics 2 has learning objectives under
Enduring Understanding 3.A that focus
on electric and magnetic forces and other
forces arising in the context of interactions
introduced in Physics 2, rather than the
mechanical systems introduced in Physics 1.
3.A.3.1 3.A.3
Analyze a scenario and make A force exerted on an object is always
claims (develop arguments, due to the interaction of that object
justify assertions) about with another object.
the forces exerted on an a. An object cannot exert a force on itself.
object by other objects for
b. Even though an object is at rest, there
different types of forces
may be forces exerted on that object
or components of forces.
[SP 6.4, 7.2] by other objects.
3.A.4.1 3.A.4
Construct explanations of If one object exerts a force on a second object,
physical situations involving the second object always exerts a force of
the interaction of bodies equal magnitude on the first object in the
using Newton’s third law and opposite direction.
the representation of action-
reaction pairs of forces.
[SP 1.4, 6.2]
3.A.4.2
Use Newton’s third law to
make claims and predictions
about the action-reaction
pairs of forces when two
objects interact. [SP 6.4, 7.2]
3.A.4.3
Analyze situations involving
interactions among several
objects by using free-body
diagrams that include the
application of Newton’s third
law to identify forces. [SP 1.4]
UNIT 4
Energy
20–28%
AP EXAM WEIGHTING
~22–25
CLASS PERIODS
Energy
Unit Overview
BIG IDEA 3 In Unit 4, students will be introduced to the idea of conservation as a foundational model
Force Interactions INT of physics, along with the concept of work as the agent of change for energy. As in earlier
§ How does pushing units, students will once again utilize both familiar and new models and representations to
something analyze physical situations, now with force or energy as major components. Students will
give it energy? be encouraged to call upon their knowledge of Units 1–4 to determine the most appropriate
technique and will be challenged to understand the limiting factors of each. Describing,
BIG IDEA 4 creating, and using these representations will also help students grapple with common
Change CHA misconceptions that they may have about energy, such as whether or not a single object
§ How is energy can “have” potential energy. A thorough understanding of these energy models will support
exchanged and students’ ability to make predications—and ultimately justify claims with evidence—about
transformed within or physical situations. This is crucial, as the mathematical models and representations used in
between systems? Unit 4 will mature throughout the course and appear in subsequent units.
§ How does the choice of As students’ comprehension of energy (particularly kinetic, potential, and microscopic internal
system influence how energy) evolves, they will begin to connect and relate knowledge across scales, concepts, and
energy is stored or how representations, as well as across disciplines, particularly physics, chemistry, and biology.
work is done?
§ How does energy
Preparing for the AP Exam
conservation allow the
riders in the back car of When students work with mathematical representations, it’s crucial that they understand the
a rollercoaster to have a connections between the mathematical description, physical phenomena, and the concepts
thrilling ride? represented in those mathematical descriptions. On the exam, students need to be able to
justify why using a particular equation to analyze a situation is useful and be aware of the
BIG IDEA 5 conditions under which equations/mathematical representations can be used. Familiarity
Conservation CON with symbolic solutions is also necessary, because students will not often encounter a
§ How can the idea of question that asks them to directly solve for a numerical answer. Finally, students need to
potential energy be be able to evaluate equations in terms of units and limiting case analysis. The exam asks
used to describe the students to translate between functional relationships in equations (proportionalities, inverse
work done to move proportionalities, etc.) and cause-and-effect relationships in the physical world.
celestial bodies?
§ How is energy
transferred between
objects or systems?
§ How does the law of
conservation of energy
govern the interactions
between objects
and systems?
4 Energy
UNIT AT A GLANCE
Understanding
Class Periods
Enduring
Work and
4.2 1.4 The student can use representations and models to analyze
Mechanical Energy situations or solve problems qualitatively and quantitatively.
2.1 The student can justify the selection of a mathematical
routine to solve problems.
2.2 The student can apply mathematical routines to
3.E, 4.C
Conservation
4.3 1.4 The student can use representations and models to analyze
of Energy, the situations or solve problems qualitatively and quantitatively.
Work-Energy 1.5 The student can re-express key elements of natural
Principle, and Power
phenomena across multiple representations in the domain.
2.1 The student can justify the selection of a mathematical
routine to solve problems.
2.2 The student can apply mathematical routines to
quantities that describe natural phenomena.
5.B
4 Energy
Making Connections
7.2
The student can connect
concepts in and across
Required Course Content domain(s) to generalize or
extrapolate in and/or across
enduring understandings
and/or big ideas.
ENDURING UNDERSTANDING
5.A
Certain quantities are conserved, in the sense that the changes of those quantities
in a given system are always equal to the transfer of that quantity to or from the
system by all possible interactions with other systems.
5.A.2.1 5.A.2
Define open and closed For all systems under all circumstances,
systems for everyday energy, charge, linear momentum, and
situations and apply angular momentum are conserved. For an
conservation concepts isolated or a closed system, conserved
for energy, charge, and quantities are constant. An open system is
linear momentum to those one that exchanges any conserved quantity
situations. [SP 6.4, 7.2] with its surroundings.
4 Energy
SCIENCE PRACTICES
Modeling
TOPIC 4.2
1.4
The student can use
representations and models
Work and
to analyze situations or
solve problems qualitatively
and quantitatively.
Mechanical Energy
Mathematical
Routines
2.1
The student can justify the
selection of a mathematical Required Course Content
routine to solve problems.
2.2
The student can apply
mathematical routines to
ENDURING UNDERSTANDING
quantities that describe
natural phenomena. 3.E
A force exerted on an object can change the kinetic energy of the object.
Argumentation
6.4
The student can make
LEARNING OBJECTIVE ESSENTIAL KNOWLEDGE
claims and predictions
about natural phenomena 3.E.1.1 3.E.1
based on scientific theories Make predictions about The change in the kinetic energy of an object
and models. the changes in kinetic depends on the force exerted on the object
energy of an object based and on the displacement of the object during
Making Connections
on considerations of the the interval that the force is exerted.
7.2 direction of the net force a. Only the component of the net force exerted
The student can connect on the object as the object on an object parallel or antiparallel to the
concepts in and across moves. [SP 6.4, 7.2]
domain(s) to generalize or
displacement of the object will increase
extrapolate in and/or across 3.E.1.2 (parallel) or decrease (antiparallel) the kinetic
enduring understandings Use net force and velocity energy of the object.
and/or big ideas. vectors to determine b. The magnitude of the change in the kinetic
qualitatively whether the energy is the product of the magnitude of
kinetic energy of an object the displacement and of the magnitude
would increase, decrease, or of the component of force parallel or
remain unchanged. [SP 1.4] antiparallel to the displacement.
3.E.1.3 Relevant Equation:
Use force and velocity ΔE = W = Fd ||
vectors to determine
qualitatively or quantitatively c. The component of the net force exerted on
the net force exerted on an object perpendicular to the direction of
an object and qualitatively the displacement of the object can change
whether the kinetic energy the direction of the motion of the object
of that object would without changing the kinetic energy of the
increase, decrease, or remain object. This should include uniform circular
unchanged. [SP 1.4, 2.2] motion and projectile motion.
2
ΔE = W = Fd = Fd cosθ
||
ENDURING UNDERSTANDING
4.C
Interactions with other objects or systems can change the total energy of a system.
4 Energy
system in linear motion will b. Work (change in energy) can be found from
change the kinetic energy the area under a graph of the magnitude
of the center of mass of the of the force component parallel to the
system, the potential energy displacement versus displacement.
of the systems, and/or the
Relevant Equation:
internal energy of the system.
[SP 1.4, 2.2, 7.2] ΔE = W = Fd = Fd cosθ
||
Mathematical
Routines
2.1
ENDURING UNDERSTANDING
The student can justify the
5.B selection of a mathematical
The energy of a system is conserved. routine to solve problems.
2.2
The student can apply
mathematical routines to
LEARNING OBJECTIVE ESSENTIAL KNOWLEDGE
quantities that describe
5.B.1.1 5.B.1 natural phenomena.
Create a representation Classically, an object can only have kinetic
or model showing that a energy since potential energy requires an Experimental
single object can only have interaction between two or more objects. Method
kinetic energy and use Relevant Equation: 4.2
information about that object 1 The student can design
to calculate its kinetic energy. K = mv 2 a plan for collecting data
[SP 1.4, 2.2] 2 to answer a particular
scientific question.
5.B.1.2 BOUNDARY STATEMENT:
Translate between a Conservation principles apply in the context Data Analysis
representation of a single of the appropriate Physics 1 and Physics 2 5.1
object, which can only have
courses. Work, potential energy, and kinetic The student can analyze
kinetic energy, and a system data to identify patterns
energy concepts are related to mechanical
that includes the object, or relationships.
which may have both kinetic systems in Physics 1 and electric, magnetic,
and potential energies. thermal, and atomic and elementary particle Argumentation
[SP 1.5] systems in Physics 2.
6.4
The student can make
claims and predictions
continued on next page about natural phenomena
based on scientific theories
and models.
Making Connections
7.2
The student can connect
concepts in and across
domain(s) to generalize or
extrapolate in and/or across
enduring understandings
and/or big ideas.
4 Energy
5.B.3.1 5.B.3
Describe and make A system with internal structure can have
qualitative and/or quantitative potential energy. Potential energy exists within
predictions about everyday a system if the objects within that system
examples of systems with interact with conservative forces.
internal potential energy. a. The work done by a conservative force is
[SP 2.2, 6.4, 7.2] independent of the path taken. The work
5.B.3.2 description is used for forces external to
Make quantitative the system. Potential energy is used when
calculations of the internal the forces are internal interactions between
potential energy of a system parts of the system.
from a description or diagram b. Changes in the internal structure can result
of that system. [SP 1.4, 2.2] in changes in potential energy. Examples
5.B.3.3 include mass-spring oscillators and objects
Apply mathematical falling in a gravitational field.
reasoning to create a c. The change in electric potential in a
description of the internal circuit is the change in potential energy
potential energy of a system per unit charge. [In Physics 1, only in the
from a description or context of circuits]
diagram of the objects and
Relevant Equations:
interactions in that system.
[SP 1.4, 2.2] l
Tp = 2π
g
Ts = 2π m
k
U s = 1 kx 2
2
ΔU g = mg Δy
5.B.5.1 5.B.5
Design an experiment and Energy can be transferred by an external force
analyze data to determine exerted on an object or a system that moves
how a force exerted on an the object or system through a distance;
object or system does work this energy transfer is called work. Energy
on the object or system as transfer in mechanical or electrical systems
it moves through a distance. may occur at different rates. Power is defined
[SP 4.2, 5.1] as the rate of energy transfer into, out of,
5.B.5.2
or within a system. [A piston filled with gas
getting compressed or expanded is treated in
Design an experiment and
Physics 2 as part of thermodynamics.]
analyze graphical data in
which interpretations of the Relevant Equations:
area under a force-distance ΔE = W = Fd = Fd cosθ
||
4 Energy
UNIT 5
Momentum
12–18%
AP EXAM WEIGHTING
~14–17
CLASS PERIODS
Momentum
Unit Overview
BIG IDEA 3 Unit 5 introduces students to the relationship between force, time, and momentum via
Force Interactions INT calculations, data analysis, designing experiments, and making predictions. Students will
§ How does pushing learn how to use new models and representations to illustrate the law of the conservation
an object change of momentum of objects and systems while simultaneously building on their knowledge
its momentum? of previously studied representations. Using the law of the conservation of momentum
to analyze physical situations gives students a more complete picture of forces and leads
BIG IDEA 4 them to revisit their misconceptions surrounding Newton’s third law. Students will also
Change CHA
have the opportunity to make connections between the conserved quantities of momentum
§ How do interactions with and energy to determine under what conditions each quantity is conserved. It’s essential
other objects or systems that students are not only comfortable solving numerical equations (such as the speed
change the linear of a system after an inelastic collision) but also confident in their ability to discuss when
momentum of a system? momentum is conserved and how the type of collision affects the outcome. Threading such
§ How is the physics connections between physical quantities is fundamental to understanding the broader
definition of momentum relationship between this unit and the rest of the course.
different from how
momentum is used
Students will have more opportunities to apply conservation laws to make predictions and
to describe things in
justify claims in Unit 7 when they are introduced to rotational quantities.
everyday life?
Preparing for the AP Exam
BIG IDEA 5
Conservation CON Physicists often use models and representations to show the design or workings of a system,
§ How does the law of an object, or a concept. Representations and models include, but are not limited to, sketching
the conservation of the physical situation, free-body diagrams, graphs, mathematical equations, and narratives.
momentum govern Unit 5 focuses on the creation/use and re-representation of models and representations.
interactions between Students should be presented with multiple opportunities to create, use, and re-represent
objects or systems? models and representations, including non-traditional representations. For example, while it is
§ How can momentum be important that students be able to create a force versus time graph and explain that the area
used to determine fault under the curve is equal to the momentum, they also need to feel comfortable with sketching or
in car crashes? analyzing a graph of momentum versus time where the slope of the line is the net external force.
In every situation, students need to be able to think about possible re-representations and how
the new representation would change the model and/or introduce new data for analysis.
5 Momentum
UNIT AT A GLANCE
Understanding
Class Periods
Enduring
Momentum and
5.1 2.1 The student can justify the selection of a mathematical
Impulse routine to solve problems.
4.1 The student can justify the selection of the kind of data
needed to answer a particular scientific question.
Representations
5.2 1.4 The student can use representations and models
of Changes to analyze situations or solve problems qualitatively
in Momentum and quantitatively.
*Indicates a science practice not assessed with its paired topic on this unit’s Personal Progress Check.
Class Periods
Enduring
Conservation of
5.4 2.1 The student can justify the selection of a mathematical
Linear Momentum routine to solve problems.
4.1 The student can justify the selection of the kind of data
needed to answer a particular scientific question.
*Indicates a science practice not assessed with its paired topic on this unit’s Personal Progress Check.
5 Momentum
Momentum and
Routines
2.1
The student can justify the
Experimental
Method
4.1
The student can justify
the selection of the
Required Course Content kind of data needed
to answer a particular
scientific question.
4.2
Data Analysis
5.1
LEARNING OBJECTIVE ESSENTIAL KNOWLEDGE
The student can analyze
3.D.1.1 3.D.1 data to identify patterns
Justify the selection of The change in momentum of an object is a or relationships.
data needed to determine vector in the direction of the net force exerted
the relationship between on the object. Argumentation
the direction of the force Relevant Equation: 6.4
acting on an object and the The student can make
change in momentum caused
p = mv claims and predictions
by that force. [SP 4.1] about natural phenomena
based on scientific theories
and models.
3.D.2.1 3.D.2
Justify the selection of The change in momentum of an object occurs
routines for the calculation over a time interval.
of the relationships a. The force that one object exerts on a
between changes in second object changes the momentum of
momentum of an object, the second object (in the absence of other
average force, impulse, and
forces on the second object).
time of interaction. [SP 2.1]
b. The change in momentum of that
3.D.2.2
object depends on the impulse, which
Predict the change in is the product of the average force
momentum of an object from and the time interval during which
the average force exerted on the interaction occurred.
the object and the interval
of time during which the Relevant Equation:
force is exerted. [SP 6.4] p = mv
5 Momentum
Representations of 1.4
The student can use
representations and models
Mathematical
Routines
2.2
The student can apply
Required Course Content mathematical routines to
quantities that describe
natural phenomena.
Data Analysis
ENDURING UNDERSTANDING 5.1
4.B The student can analyze
Interactions with other objects or systems can change the total linear momentum data to identify patterns
of a system. or relationships.
5 Momentum
AP Physics 1: Algebra-Based Course and Exam Description Course Framework V.1 | 100
Return to Table of Contents
© 2021 College Board
UNIT
Momentum
5
TOPIC 5.3 SCIENCE PRACTICES
Argumentation
Making Connections
7.2
The student can connect
concepts in and across
Required Course Content domain(s) to generalize or
extrapolate in and/or across
enduring understandings
and/or big ideas.
ENDURING UNDERSTANDING
5.A
Certain quantities are conserved, in the sense that the changes of those quantities
in a given system are always equal to the transfer of that quantity to or from the
system by all possible interactions with other systems.
AP Physics 1: Algebra-Based Course and Exam Description Course Framework V.1 | 101
Return to Table of Contents
© 2021 College Board
UNIT
5 Momentum
SCIENCE PRACTICES
Mathematical
TOPIC 5.4
Conservation of
Routines
2.1
The student can justify the
selection of a mathematical
routine to solve problems.
2.2
Linear Momentum
The student can apply
mathematical routines to
quantities that describe
natural phenomena.
AP Physics 1: Algebra-Based Course and Exam Description Course Framework V.1 | 102
Return to Table of Contents
© 2021 College Board
UNIT
Momentum
5
SCIENCE PRACTICES
(CONT’D)
LEARNING OBJECTIVE ESSENTIAL KNOWLEDGE Argumentation
5.D.1.3
BOUNDARY STATEMENT: 6.4
Apply mathematical routines The student can make
Physics 1 includes a quantitative and
appropriately to problems claims and predictions
involving elastic collisions qualitative treatment of conservation
about natural phenomena
in one dimension and justify of momentum in one dimension and a based on scientific theories
the selection of those semiquantitative treatment of conservation and models.
mathematical routines of momentum in two dimensions. Test items
based on conservation of involving solution of simultaneous equations Making Connections
momentum and restoration of are not included in Physics 1, but items 7.2
kinetic energy. [SP 2.1, 2.2] testing whether students can set up the The student can connect
5.D.1.4 equations properly and can reason about how concepts in and across
Design an experimental changing a given mass, speed, or angle would domain(s) to generalize or
test of an application of the extrapolate in and/or across
affect other quantities are included.
enduring understandings
principle of the conservation
Physics 1 includes only conceptual and/or big ideas.
of linear momentum, predict
understanding of the center of mass motion
an outcome of the experiment
using the principle, of a system without the need for calculation
analyze data generated of center of mass.
by that experiment whose The Physics 1 course includes topics from
uncertainties are expressed Enduring Understanding 5.D in the context
numerically, and evaluate of mechanical systems.
the match between the
prediction and the outcome.
[SP 4.2, 5.1, 5.3, 6.4]
5.D.1.5
Classify a given collision
situation as elastic or
inelastic, justify the selection
of conservation of linear
momentum and restoration
of kinetic energy as the
appropriate principles for
analyzing an elastic collision,
solve for missing variables,
and calculate their values.
[SP 2.1, 2.2]
5.D.2.1 5.D.2
Qualitatively predict, in In a collision between objects, linear momentum is
terms of linear momentum conserved. In an inelastic collision, kinetic energy
and kinetic energy, how is not the same before and after the collision.
the outcome of a collision a. In a closed system, the linear momentum is
between two objects constant throughout the collision.
changes depending
b. In a closed system, the kinetic energy after
on whether the collision
an inelastic collision is different from the
is elastic or inelastic.
kinetic energy before the collision.
[SP 6.4, 7.2]
Relevant Equations:
p = mv
K = 1 mv 2
2
5 Momentum
AP Physics 1: Algebra-Based Course and Exam Description Course Framework V.1 | 104
Return to Table of Contents
© 2021 College Board
UNIT
Momentum
5
LEARNING OBJECTIVE ESSENTIAL KNOWLEDGE
5.D.3.1 5.D.3
Predict the velocity of the The velocity of the center of mass of the
center of mass of a system system cannot be changed by an interaction
when there is no interaction within the system. [Physics 1 includes no
outside of the system but calculations of centers of mass; the equation is
there is an interaction within not provided until Physics 2. However, without
the system (i.e., the student doing calculations, Physics 1 students are
simply recognizes that expected to be able to locate the center of
interactions within a system do mass of highly symmetric mass distributions,
not affect the center-of-mass such as a uniform rod or cube of uniform
motion of the system and is density, or two spheres of equal mass.]
able to determine that there is a. The center of mass of a system depends
no external force). [SP 6.4] on the masses and positions of the objects
in the system. In an isolated system (a system
with no external forces), the velocity of the
center of mass does not change.
b. When objects in a system collide, the
velocity of the center of mass of the system
will not change unless an external force is
exerted on the system.
c. Included in Physics 1 is the idea that, where
there is both a heavier and lighter mass,
the center of mass is closer to the heavier
mass. Only a qualitative understanding of
this concept is required.
AP Physics 1: Algebra-Based Course and Exam Description Course Framework V.1 | 105
Return to Table of Contents
© 2021 College Board
THIS PAGE IS INTENTIONALLY LEFT BLANK.
AP PHYSICS 1
UNIT 6
Simple
Harmonic
Motion
4–6%
AP EXAM WEIGHTING
~4–7
CLASS PERIODS
AP Physics 1: Algebra-Based Course and Exam Description Course Framework V.1 | 107
Return to Table of Contents
© 2021 College Board
Remember to go to AP Classroom
to assign students the online
Personal Progress Check for
this unit.
Whether assigned as homework or
completed in class, the Personal
Progress Check provides each
student with immediate feedback
related to this unit’s topics and
science practices.
AP Physics 1: Algebra-Based Course and Exam Description Course Framework V.1 | 108
Return to Table of Contents
© 2021 College Board
UNIT
Simple Harmonic
Motion
Unit Overview
BIG IDEA 3 In Unit 6, students will continue to use the same tools, techniques, and models that they
Force Interactions INT have been using throughout this course. However, they will now use them to analyze
§ How does a restoring a new type of motion: simple harmonic motion. Although simple harmonic motion is unique,
force differ from a students will learn that even in new situations, the fundamental laws of physics remain the
“regular” force? same. Energy bar charts, as well as free-body diagrams, become increasingly important
§ How does the presence
as students work toward determining which model is most appropriate for a given physical
of restoring forces
situation. Preconceptions—such as the relationship between the amplitude and period of
predict and lead to oscillation—will also be addressed to provide students with a more nuanced awareness of
harmonic motion? simple harmonic motion.
§ How does a Students are expected to use the content knowledge they gained in the first five units to
spring cause an make and defend claims while also making connections in and across the content topics
object to oscillate? and big ideas. Because Unit 6 is the first unit in which students possess all the tools of force,
§ How can oscillations energy, and momentum conservation, it’s important that teachers scaffold lessons to help
be used to make them develop a better understanding of each fundamental physics principles as well as its
our lives easier? limitations. Throughout this unit, students will be asked to create force, energy, momentum,
and position versus time graphs for a single scenario and to make predictions based on their
BIG IDEA 5 representations. Students will enhance their study of motion when they learn about oscillatory
Conservation CON motion in Unit 10.
§ How does the law
of conservation of Preparing for the AP Exam
energy govern the
interactions between Some questions on AP Physics 1 Exam require students to identify more than one correct
objects and systems? answer. Because these multiple-select questions can easily intimidate students, we highly
recommend that they take the time to read the entire prompt. Students who jump right
§ How can energy stored
to the answers could be frustrated to find answer choices that are factually correct but
in a spring be used to
do not complete the task. Remember: Students will only get credit if they choose all the
create motion?
correct answers.
AP Physics 1: Algebra-Based Course and Exam Description Course Framework V.1 | 109
Return to Table of Contents
© 2021 College Board
UNIT
UNIT AT A GLANCE
Understanding
Class Periods
Enduring
Period of Simple
6.1 2.2 The student can apply mathematical routines to
Harmonic quantities that describe natural phenomena.
Oscillators
4.2 The student can design a plan for collecting data to
answer a particular scientific question.
Energy of a Simple
6.2 1.4 The student can use representations and models to analyze
Harmonic Oscillator situations or solve problems qualitatively and quantitatively.
AP Physics 1: Algebra-Based Course and Exam Description Course Framework V.1 | 110
Return to Table of Contents
© 2021 College Board
UNIT
Simple Harmonic Motion
6
SAMPLE INSTRUCTIONAL ACTIVITIES
The sample activities on this page are optional and are offered to provide possible ways to
incorporate various instructional approaches the classroom. Teachers do not need to use
these activities or instructional approaches and are free to alter or edit them. The examples
below were developed in partnership with teachers from the AP community to share ways
that they approach teaching some of the topics in this unit. Please refer to the Instructional
Approaches section beginning on p. 139 for more examples of activities and strategies.
AP Physics 1: Algebra-Based Course and Exam Description Course Framework V.1 | 111
Return to Table of Contents
© 2021 College Board
UNIT
SCIENCE PRACTICES
Mathematical
TOPIC 6.1
Period of Simple
Routines
2.2
The student can apply
mathematical routines to
quantities that describe
natural phenomena.
Harmonic Oscillators
Experimental
Method
4.2
The student can design
a plan for collecting data Required Course Content
to answer a particular
scientific question.
Data Analysis
5.1
ENDURING UNDERSTANDING
The student can analyze 3.B
data to identify patterns Classically, the acceleration of an object interacting with other objects can be predicted
or relationships.
F
by using a = .
Argumentation m
6.2
The student can LEARNING OBJECTIVE ESSENTIAL KNOWLEDGE
construct explanations
of phenomena based on 3.B.3.1 3.B.3
evidence produced through Predict which properties Restoring forces can result in oscillatory motion.
scientific practices. determine the motion of a When a linear restoring force is exerted on an
6.4 simple harmonic oscillator and object displaced from an equilibrium position,
The student can make what the dependence of the the object will undergo a special type of motion
claims and predictions motion is on those properties. called simple harmonic motion. Examples
about natural phenomena [SP 6.4, 7.2] include gravitational force exerted by Earth on a
based on scientific theories simple pendulum and mass-spring oscillator.
and models. 3.B.3.2
a. For a spring that exerts a linear restoring
Design a plan and collect force, the period of a mass-spring oscillator
Making Connections data in order to ascertain the increases with mass and decreases with
7.2 characteristics of the motion spring stiffness.
The student can connect of a system undergoing
b. For a simple pendulum, the period increases
concepts in and across oscillatory motion caused by a
with the length of the pendulum and decreases
domain(s) to generalize or restoring force. [SP 4.2]
with the magnitude of the gravitational field.
extrapolate in and/or across
enduring understandings 3.B.3.3 c. Minima, maxima, and zeros of position,
and/or big ideas. Analyze data to identify velocity, and acceleration are features of
qualitative and quantitative harmonic motion. Students should be able
relationships between given to calculate force and acceleration for any
values and variables (i.e., force, given displacement for an object oscillating
displacement, acceleration, on a spring.
velocity, period of motion, Relevant Equations:
frequency, spring constant,
string length, mass) associated l
Tp = 2π
with objects in oscillatory g
motion and use those data
to determine the value of an m
Ts = 2π
unknown. [SP 2.2, 5.1] k
AP Physics 1: Algebra-Based Course and Exam Description Course Framework V.1 | 113
Return to Table of Contents
© 2021 College Board
UNIT
SCIENCE PRACTICES
Modeling
TOPIC 6.2
1.4
The student can use
representations and models
Energy of a Simple
to analyze situations or
solve problems qualitatively
and quantitatively.
Harmonic Oscillator
Mathematical
Routines
2.1
The student can justify the
selection of a mathematical Required Course Content
routine to solve problems.
2.2
The student can apply
mathematical routines to
ENDURING UNDERSTANDING
quantities that describe
natural phenomena. 5.B
The energy of a system is conserved.
Argumentation
6.4
The student can make
LEARNING OBJECTIVE ESSENTIAL KNOWLEDGE
claims and predictions
about natural phenomena 5.B.2.1 5.B.2
based on scientific theories Calculate the expected A system with internal structure can have
and models. behavior of a system internal energy, and changes in a system’s
using the object model internal structure can result in changes in
Making Connections
(i.e., by ignoring changes internal energy. [Physics 1 includes mass-
7.2 in internal structure) to spring oscillators and simple pendulums.
The student can connect analyze a situation. Then, Physics 2 includes charged objects in electric
concepts in and across when the model fails, the fields and examining changes in internal energy
domain(s) to generalize or student can justify the use with changes in configuration.]
extrapolate in and/or across of conservation of energy
enduring understandings
principles to calculate the
and/or big ideas.
change in internal energy
due to changes in internal
structure because the
object is actually a system.
[SP 1.4, 2.1]
AP Physics 1: Algebra-Based Course and Exam Description Course Framework V.1 | 114
Return to Table of Contents
© 2021 College Board
UNIT
Simple Harmonic Motion
6
LEARNING OBJECTIVE ESSENTIAL KNOWLEDGE
5.B.3.1 5.B.3
Describe and make A system with internal structure can have
qualitative and/or quantitative potential energy. Potential energy exists within
predictions about everyday a system if the objects within that system
examples of systems with interact with conservative forces.
internal potential energy. a. The work done by a conservative force is
[SP 2.2, 6.4, 7.2] independent of the path taken. The work
5.B.3.2 description is used for forces external to
Make quantitative the system. Potential energy is used when
calculations of the internal the forces are internal interactions between
potential energy of a system parts of the system.
from a description or diagram b. Changes in the internal structure can result
of that system. [SP 1.4, 2.2] in changes in potential energy. Examples
5.B.3.3 include mass-spring oscillators and objects
Apply mathematical falling in a gravitational field.
reasoning to create a c. The change in electric potential in a
description of the internal circuit is the change in potential energy
potential energy of a system per unit charge. [In Physics 1, only in the
from a description or context of circuits]
diagram of the objects and
Relevant Equations:
interactions in that system.
[SP 1.4, 2.2] l
Tp = 2π
g
Ts = 2π m
k
1
U s = kx 2
2
ΔU g = mg Δy
5.B.4.1 5.B.4
Describe and make The internal energy of a system includes
predictions about the the kinetic energy of the objects that make
internal energy of systems. up the system and the potential energy of
[SP 6.4, 7.2] the configuration of the objects that make
5.B.4.2
up the system.
Calculate changes in kinetic a. Since energy is constant in a closed
energy and potential energy system, changes in a system’s potential
of a system using information energy can result in changes to the
from representations of that system’s kinetic energy.
system. [SP 1.4, 2.1, 2.2] b. The changes in potential and kinetic
energies in a system may be further
constrained by the construction
of the system.
AP Physics 1: Algebra-Based Course and Exam Description Course Framework V.1 | 115
Return to Table of Contents
© 2021 College Board
THIS PAGE IS INTENTIONALLY LEFT BLANK.
AP PHYSICS 1
UNIT 7
Torque and
Rotational
Motion
12–18%
AP EXAM WEIGHTING
~14–19
CLASS PERIODS
AP Physics 1: Algebra-Based Course and Exam Description Course Framework V.1 | 117
Return to Table of Contents
© 2021 College Board
Remember to go to AP Classroom
to assign students the online
Personal Progress Check for
this unit.
Whether assigned as homework or
completed in class, the Personal
Progress Check provides each
student with immediate feedback
related to this unit’s topics and
science practices.
AP Physics 1: Algebra-Based Course and Exam Description Course Framework V.1 | 118
Return to Table of Contents
© 2021 College Board
UNIT
Torque and
Rotational Motion
Unit Overview
BIG IDEA 3 Unit 7 completes the study of mechanical physics by introducing students to torque
Force Interactions INT and rotational motion. Although these topics present more complex scenarios, the tools
§ How does a system at of analysis remain the same: The content and models explored in the first six units of
rotational equilibrium AP Physics 1 set the foundation for Unit 7.
compare to a system in
During their study of torque and rotational motion, students will be confronted with different
translational equilibrium?
ways of thinking about and modeling forces. As in previous units, it’s critical that students are
§ How does the choice given opportunities to create and use representations and models to make predictions and
of system and rotation justify claims. It’s equally important that students are comfortable deriving new expressions
point affect the forces from fundamental principles to help them make predictions in unfamiliar, applied contexts.
that can cause a torque
on an object or a system? Unit 7 also focuses on the mathematical practice of estimating quantities that can describe
§ How can balanced
natural phenomena. For example, students need to be able to estimate the torque on an
forces cause rotation?
object caused by various forces in comparison to other situations. Although this particular
science practice doesn’t appear often in AP Physics 1, it nonetheless is an important
§ Why does it matter
conceptual skill for students to be able to compare estimated values of physical quantities.
where the door
handle is placed? Throughout this unit, students will have opportunities to compare and connect their
§ Why are long wrenches understanding of linear and rotational motion, dynamics, energy, and momentum to make
more effective? meaning of these concepts as a whole, rather than as distinct and separate units.
BIG IDEA 4
Change CHA Preparing for the AP Exam
§ How can an external Students must be familiar with identifying and analyzing functional relationships, especially
net torque change the with equations that are not found on the equation sheet. It is likely that students will be
angular momentum asked to explain why a new equation—one that they have never seen before—does or does
of a system? not support a claim. They may also have to briefly address functional dependence in
§ Why is a rotating bicycle questions. Students will be more prepared for the type of mathematical reasoning required
wheel more stable than on the AP Physics 1 Exam if they understand and practice making logical mathematical
a stationary one? derivations while showing their starting principle and annotating their steps.
BIG IDEA 5
Conservation CON
§ How does the
conservation of angular
momentum govern
interactions between
objects and systems?
§ Why do planets move
faster when they travel
closer to the sun?
AP Physics 1: Algebra-Based Course and Exam Description Course Framework V.1 | 119
Return to Table of Contents
© 2021 College Board
UNIT
UNIT AT A GLANCE
Understanding
Class Periods
Enduring
Rotational
7.1 1.5 The student can re-express key elements of natural
Kinematics phenomena across multiple representations in the domain.
4.1 The student can justify the selection of the kind of data
needed to answer a particular scientific question.
3.F
*Indicates a science practice not assessed with its paired topic on this unit’s Personal Progress Check.
AP Physics 1: Algebra-Based Course and Exam Description Course Framework V.1 | 120
Return to Table of Contents
© 2021 College Board
UNIT
Torque and Rotational Motion
7
UNIT AT A GLANCE (cont’d)
Understanding
Class Periods
Enduring
Angular Momentum
7.3 1.2 The student can describe representations and models of
and Torque natural or man-made phenomena and systems in the domain.*
4.1 The student can justify the selection of the kind of data
needed to answer a particular scientific question.
Conservation of
7.4 2.1 The student can justify the selection of a mathematical
Angular Momentum routine to solve problems.
*Indicates a science practice not assessed with its paired topic on this unit’s Personal Progress Check.
AP Physics 1: Algebra-Based Course and Exam Description Course Framework V.1 | 121
Return to Table of Contents
© 2021 College Board
UNIT
4 7.3 Ranking
A wheel rolls down an incline from rest and across a flat surface. Case 1: Tracks are rough
enough that there is no slipping. Case 2: Tracks have some friction, but there is slipping.
Case 3: Tracks have negligible friction. Have students rank translational kinetic energies at
the end, rotational kinetic energies at the end, and total mechanical energies of the wheel
at the end as three separate tasks. (KT3 > KT2 > KT1), (KR1 > KR2 > KR3), and (E1 = E3 > E2).
AP Physics 1: Algebra-Based Course and Exam Description Course Framework V.1 | 122
Return to Table of Contents
© 2021 College Board
UNIT
Torque and Rotational Motion
7
TOPIC 7.1 SCIENCE PRACTICES
Modeling
Mathematical
Routines
2.1
The student can justify the
Required Course Content selection of a mathematical
routine to solve problems.
2.2
The student can apply
mathematical routines to
ENDURING UNDERSTANDING
quantities that describe
3.A natural phenomena.
All forces share certain common characteristics when considered by observers in inertial
reference frames.
AP Physics 1: Algebra-Based Course and Exam Description Course Framework V.1 | 123
Return to Table of Contents
© 2021 College Board
UNIT
BOUNDARY STATEMENT:
AP Physics 2 has learning objectives under
Enduring Understanding 3.A that focus
on electric and magnetic forces and other
forces arising in the context of interactions
introduced in Physics 2, rather than the
mechanical systems introduced in Physics 1.
AP Physics 1: Algebra-Based Course and Exam Description Course Framework V.1 | 124
Return to Table of Contents
© 2021 College Board
UNIT
Torque and Rotational Motion
7
TOPIC 7.2 SCIENCE PRACTICES
Modeling
Mathematical
Routines
2.1
The student can justify the
Required Course Content selection of a mathematical
routine to solve problems.
2.2
The student can apply
mathematical routines to
ENDURING UNDERSTANDING
quantities that describe
3.F natural phenomena.
A force exerted on an object can cause a torque on that object. 2.3
The student can estimate
quantities that describe
natural phenomena.
LEARNING OBJECTIVE ESSENTIAL KNOWLEDGE
3.F.1.1 3.F.1 Experimental
Use representations of the Only the force component perpendicular to Method
relationship between force the line connecting the axis of rotation and the 4.1
and torque. [SP 1.4] point of application of the force results in a The student can justify
3.F.1.2
torque about that axis. the selection of the
a. The lever arm is the perpendicular distance kind of data needed
Compare the torques on an
to answer a particular
object caused by various from the axis of rotation or revolution to the
scientific question.
forces. [SP 1.4] line of application of the force.
4.2
3.F.1.3
b. The magnitude of the torque is the product
The student can design
of the magnitude of the lever arm and the
Estimate the torque on an a plan for collecting data
magnitude of the force.
object caused by various to answer a particular
forces in comparison with c. The net torque on a balanced system is zero. scientific question.
other situations. [SP 2.3] Relevant Equations: Data Analysis
3.F.1.4 τ = r⊥ F = rF sin θ 5.1
Design an experiment
The student can analyze
and analyze data testing a BOUNDARY STATEMENT:
data to identify patterns
question about torques in Quantities such as angular acceleration, or relationships.
a balanced rigid system. velocity, and momentum are defined as 5.3
[SP 4.1, 4.2, 5.1] vector quantities, but in Physics 1 the The student can
3.F.1.5 determination of “direction” is limited to evaluate the evidence
Calculate torques on a clockwise and counterclockwise with respect provided by data sets in
two-dimensional system to a given axis of rotation. relation to a particular
in static equilibrium by scientific question.
examining a representation
or model (such as a diagram
or physical construction).
[SP 1.4, 2.2]
AP Physics 1: Algebra-Based Course and Exam Description Course Framework V.1 | 125
Return to Table of Contents
© 2021 College Board
UNIT
SCIENCE PRACTICES
(CONT’D)
Argumentation
LEARNING OBJECTIVE ESSENTIAL KNOWLEDGE
3.F.2.1 3.F.2
6.4
The student can make Make predictions about The presence of a net torque along any axis will
claims and predictions the change in the angular cause a rigid system to change its rotational
about natural phenomena velocity about an axis for an motion or an object to change its rotational
based on scientific theories object when forces exerted motion about that axis.
and models. on the object cause a torque a. Rotational motion can be described in terms
about that axis. [SP 6.4] of angular displacement, angular velocity,
Making Connections
3.F.2.2 and angular acceleration about a fixed axis.
7.2
Plan data-collection and b. Rotational motion of a point can be related to
The student can connect
analysis strategies designed linear motion of the point using the distance
concepts in and across
domain(s) to generalize or
to test the relationship of the point from the axis of rotation.
extrapolate in and/or across between a torque exerted c. The angular acceleration of an object or
enduring understandings on an object and the a rigid system can be calculated from the
and/or big ideas. change in angular velocity
net torque and the rotational inertia of the
of that object about an axis.
object or rigid system.
[SP 4.1, 4.2, 5.1]
Relevant Equations:
τ = r⊥ F = rF sinθ
α = τ
I
θ = θ0 + ω0t + 1 α t 2
2
ω = ω0 + α t
ω 2 = ω02 + 2α (θ − θ0 )
AP Physics 1: Algebra-Based Course and Exam Description Course Framework V.1 | 126
Return to Table of Contents
© 2021 College Board
UNIT
Torque and Rotational Motion
7
LEARNING OBJECTIVE ESSENTIAL KNOWLEDGE
3.F.3.1 3.F.3
Predict the behavior of A torque exerted on an object can change the
rotational collision situations angular momentum of an object.
by the same processes that a. Angular momentum is a vector quantity, with
are used to analyze linear its direction determined by a right-hand rule.
collision situations using an
b. The magnitude of angular momentum of a
analogy between impulse and
change of linear momentum point object about an axis can be calculated
and angular impulse by multiplying the perpendicular distance
and change of angular from the axis of rotation to the line of motion
momentum. [SP 6.4, 7.2] by the magnitude of linear momentum.
c. The magnitude of angular momentum of
3.F.3.2
an extended object can also be found by
In an unfamiliar context
multiplying the rotational inertia by the
or using representations
angular velocity. Students do not need to
beyond equations, justify the
know the equation for an object’s rotational
selection of a mathematical
inertia, as it will be provided at the exam.
routine to solve for the
change in angular momentum They should have a qualitative sense of what
of an object caused by factors affect rotational inertia—for example,
torques exerted on the why a hoop has more rotational inertia than
object. [SP 2.1] a puck of the same mass and radius.
3.F.3.3
d. The change in angular momentum of an
object is given by the product of the average
Plan data-collection and
torque and the time the torque is exerted.
analysis strategies designed
to test the relationship Relevant Equations:
between torques exerted on L = Iω
an object and the change in
ΔL = τΔt
angular momentum of that
object. [SP 4.1, 4.2, 5.1, 5.3] L = mvr
AP Physics 1: Algebra-Based Course and Exam Description Course Framework V.1 | 127
Return to Table of Contents
© 2021 College Board
UNIT
SCIENCE PRACTICES
Modeling
TOPIC 7.3
1.2
The student can describe
representations and models
Angular Momentum
of natural or man-made
phenomena and systems in
the domain.
and Torque
1.4
The student can use
representations and models
to analyze situations or
solve problems qualitatively
and quantitatively. Required Course Content
Mathematical
Routines
2.2
ENDURING UNDERSTANDING
The student can apply
mathematical routines to 4.D
quantities that describe A net torque exerted on a system by other objects or systems will change the
natural phenomena. angular momentum of the system.
Scientific
Questioning
3.2
LEARNING OBJECTIVE ESSENTIAL KNOWLEDGE
The student can refine 4.D.1.1 4.D.1
scientific questions. Describe a representation Torque, angular velocity, angular acceleration,
and use it to analyze a and angular momentum are vectors and can
Experimental
situation in which several be characterized as positive or negative
Method
forces exerted on a rotating depending on whether they give rise to or
4.1 system of rigidly connected correspond to counterclockwise or clockwise
The student can justify objects change the angular rotation with respect to an axis.
the selection of the velocity and angular Relevant Equations:
kind of data needed momentum of the system.
to answer a particular
τ = r⊥ F = rF sin θ
[SP 1.2, 1.4]
scientific question. τ
4.D.1.2 α=
4.2 I
Plan data-collection
The student can design L = Iω
a plan for collecting data strategies designed to
to answer a particular establish that torque, angular ΔL = τΔt
scientific question. velocity, angular acceleration, 1
and angular momentum θ = θ0 + ω0t + α t 2
Data Analysis can be predicted accurately 2
5.1 when the variables are ω = ω0 + α t
The student can analyze treated as being clockwise ω 2 = ω02 + 2α (θ − θ0 )
data to identify patterns or counterclockwise with
or relationships. respect to a well-defined BOUNDARY STATEMENT:
axis of rotation, and refine
5.3 Students do not need to know the right-hand
the research question based
The student can rule. A full dynamic treatment of rolling
evaluate the evidence on the examination of data.
[SP 3.2, 4.1, 4.2, 5.1, 5.3] without slipping—for instance, using forces
provided by data sets in
and torques to find the linear and angular
relation to a particular
scientific question. acceleration of a cylinder rolling down a
ramp—is not included in Physics 1.
AP Physics 1: Algebra-Based Course and Exam Description Course Framework V.1 | 128
Return to Table of Contents
© 2021 College Board
UNIT
Torque and Rotational Motion
7
LEARNING OBJECTIVE ESSENTIAL KNOWLEDGE
4.D.2.1 4.D.2
Describe a model of a The angular momentum of a system may
rotational system and use change due to interactions with other objects
that model to analyze a or systems.
situation in which angular a. The angular momentum of a system with
momentum changes due to respect to an axis of rotation is the sum of
interaction with other objects the angular momenta, with respect to that
or systems. [SP 1.2, 1.4]
axis, of the objects that make up the system.
4.D.2.2 b. The angular momentum of an object about
Plan a data-collection a fixed axis can be found by multiplying
and analysis strategy to the momentum of the particle by the
determine the change in perpendicular distance from the axis to the
angular momentum of line of motion of the object.
a system and relate it to
c. Alternatively, the angular momentum of a
interactions with other
system can be found from the product of the
objects and systems. [SP 4.2]
system’s rotational inertia and its angular
velocity. Students do not need to know the
equation for an object’s rotational inertia, as
it will be provided at the exam. They should
have a qualitative sense that rotational
inertia is larger when the mass is farther
from the axis of rotation.
Relevant Equations:
L = Iω
ΔL = τΔt
τ = r⊥ F = rF sin θ
Alternatively, the angular momentum of a system
can be found from the product of the system’s
rotational inertia and its angular velocity.
Students do not need to know the equation for
an object’s rotational inertia, as it will be provided
at the exam. They should have a qualitative
sense that rotational inertia is larger when the
mass is farther from the axis of rotation.
AP Physics 1: Algebra-Based Course and Exam Description Course Framework V.1 | 129
Return to Table of Contents
© 2021 College Board
UNIT
AP Physics 1: Algebra-Based Course and Exam Description Course Framework V.1 | 130
Return to Table of Contents
© 2021 College Board
UNIT
Torque and Rotational Motion
7
TOPIC 7.4 SCIENCE PRACTICES
Mathematical
Conservation of
Routines
2.1
The student can justify the
AP Physics 1: Algebra-Based Course and Exam Description Course Framework V.1 | 131
Return to Table of Contents
© 2021 College Board
UNIT
AP Physics 1: Algebra-Based Course and Exam Description Course Framework V.1 | 132
Return to Table of Contents
© 2021 College Board
AP PHYSICS 1
Laboratory
Investigations
Lab Experiments
Although laboratory work has often been separated sophisticated physics equipment, such as air tracks
from classroom work, research shows that experience and force sensors. Remember that the AP lab should
and experiment are often more instructionally effective. provide experience for students equivalent to that
Familiarity with concrete evidence leads to a deeper of a college laboratory, so teachers are encouraged to
understanding and gives students a sense of ownership make every effort to provide a range of experiences—
with the knowledge they have constructed. from experiments students contrive from string, and
duct tape to experiments in which students gather and
AP Physics courses require students to engage with
analyze data using calculators or computer-interfaced
data in a variety of ways. The analysis, interpretation,
equipment.
and application of quantitative information are vital
skills for students. Scientific inquiry experiences in There are avenues that teachers can explore as a
AP Physics 1 should be designed and implemented with means of getting access to more expensive equipment,
increasing student involvement to help enhance inquiry such as computers and probes. Probes can often be
learning and develop critical thinking and problem- rented for short periods of time from instrument
solving skills. Typically, the level of investigations in an suppliers. Alternatively, local colleges or universities
AP Physics 1 classroom should focus primarily on the may allow high school students to complete a lab as a
continuum between guided and open inquiry. However, field trip on their campus, or they may allow teachers to
depending on students’ familiarity with the topic, a given borrow their equipment. They may even donate their old
laboratory experience might incorporate a sequence equipment. Some schools have partnerships with local
involving all four levels of inquiry (confirmation, businesses that can help with laboratory equipment and
structured inquiry, guided inquiry, and open inquiry). materials. Teachers can also utilize online donation sites
such as Donors Choose and Adopt-A-Classroom.
Lab Manuals and Lab Notebooks
College Board publishes AP Physics 1 and 2 Lab Time
Inquiry-Based Lab Investigations: A Teacher’s Manual
For this AP Physics 1 to be comparable to a college
to support the guided inquiry lab requirement for the
physics course, it is critical that teachers make
course. It includes labs that teachers can choose
laboratory work an important part their curriculum.
from to satisfy the guided inquiry lab component for
An analysis of data from AP Physics examinees,
the course. Many publishers and science classroom
regarding the length of time they spent per week
material distributors offer affordable lab manuals
in the laboratory, shows that increased laboratory
with outlined experiments and activities as well as lab
time correlates with higher AP scores. Flexible or
notebooks for recording lab data and observations.
modular scheduling must be implemented to meet
Students can use any type of notebook to fulfill the
the time requirements identified in the course outline.
lab notebook requirement, even an online document.
At minimum, one double period a week is needed.
Consider the needs of the classroom when deciding
Furthermore, it is important that the AP Physics 1
what type of lab notebook to use.
laboratory program be adapted to local conditions and
funding as it aims to offer the students a well-rounded
Lab Materials experience with experimental physics. Adequate
A wide range of equipment may be used in the physics laboratory facilities should be provided so that each
laboratory, from generic lab items, such as metersticks, student has a work space where equipment and
rubber balls, springs, string, metal spheres, calibrated materials can be left overnight if necessary. Sufficient
mass sets, beakers, electronic balances, stopwatches, laboratory equipment for the anticipated enrollment
clamps, and ring stands, to items more specific to and appropriate instruments should be provided.
physics, such as tracks and carts. Successful guided Students in AP Physics 1 should have access to
inquiry student work can be accomplished with computers with software appropriate for processing
both simple, inexpensive materials and with more laboratory data and writing reports.
AP Physics 1: Algebra-Based Course and Exam Description Laboratory Investigations V.1 | 135
Return to Table of Contents
© 2021 College Board
How to Set Up a Lab Program
AP Physics 1: Algebra-Based Course and Exam Description Laboratory Investigations V.1 | 136
Return to Table of Contents
© 2021 College Board
Communication, Group specific guidelines prior to students’ discussion on
investigation designs for each experiment so that those
Collaboration, and the precautions can be incorporated into final student-
Laboratory Record selected lab designs and included in the background or
design plan in a laboratory record. It may also be helpful
Laboratory work is an excellent means through which
to print the precautions that apply to that specific
students can develop and practice communication
lab as Safety Notes to place on the desk or wall near
skills. Success in subsequent work in physics depends
student workstations. Additionally, a general set of
heavily on an ability to communicate observations,
safety guidelines should be set forth for students at
ideas, and conclusions. Students must learn to
the beginning of the course. The following is a list of
recognize that an understanding of physics is
possible general guidelines teachers may post:
relatively useless unless they can communicate their
knowledge effectively to others. By working together §§ Before every lab, make sure you know and record
in a truly collaborative manner to plan and carry out the potential hazards involved in the investigation,
experiments, students learn oral communication skills as well as the precautions you will take to stay safe.
and teamwork. Students must be encouraged to take §§ Before using equipment, make sure you know the
full individual responsibility for the success, or failure, of proper method of use to acquire good data and
the collaboration. avoid damage to equipment.
After students are given a question for investigation, §§ Know where safety equipment is located in the lab,
they may present their findings in either a written or such as the fire extinguisher, safety goggles, and
an oral report to the teacher and class for feedback the first aid kit.
and critique on their final design and results. Students §§ Follow the teacher’s special safety guidelines as set
should be encouraged to critique and challenge one forth prior to each experiment. (Students should
another’s claims based on the evidence collected record these as part of their design plan for a lab.)
during the investigation.
§§ When in doubt about the safety or advisability
of a procedure, check with the teacher
Laboratory Safety before proceeding.
Giving students the responsibility for design of Teachers should interact constantly with students as
their own laboratory experience involves special they work to observe safety practices and anticipate
responsibilities for teachers. To ensure a safe and discuss with them any problems that may arise.
working environment, teachers should first provide Walking among student groups, asking questions, and
the limitations and safety precautions necessary for showing interest in students’ work allows teachers to
potential procedures and equipment students may use keep the pulse of what students are doing and maintain
during their investigation. Teachers should also provide a watchful eye for potential safety issues.
AP Physics 1: Algebra-Based Course and Exam Description Laboratory Investigations V.1 | 137
Return to Table of Contents
© 2021 College Board
THIS PAGE IS INTENTIONALLY LEFT BLANK.
AP PHYSICS 1
Instructional
Approaches
Selecting and Using
Course Materials
Teachers will benefit from a wide array of materials AP Central provides an example textbook list to help
to help students become proficient with the science determine whether a text is considered appropriate
practices necessary to develop a conceptual in meeting the AP Physics 1 Course Audit resource
understanding of the relationships, laws, and requirement. Teachers can also select textbooks locally.
phenomena studied in AP Physics 1. In addition
to using a college-level textbook that will provide
AP Physics 1 Student Workbook
required course content, students should have regular
opportunities to create and use data, representations, The AP Physics 1 Student Workbook is a resource
and models. Rich, experimental investigation is the published by College Board to help students further
cornerstone of AP Physics 1, and diverse source their understanding of the science practices and
material allows teachers more flexibility in designing content needed to be successful on the AP Physics 1
the types of learning activities that will help develop the Exam. The workbook is scaffolded within each unit
habits of thinking like a physicist. and across the course to help students access the
material. Students will be challenged to analyze
scenarios that progress in difficulty. Each page is
Textbooks tagged to specific essential knowledge statements and
While nearly all college-level physics textbooks address science practices that students will use complete the
the 10 units of AP Physics 1, it’s important for teachers assignment. The workbook also contains resources
to identify other types of secondary sources to for both new and experienced AP teachers on how to
supplement the chosen textbook accordingly, ensuring prepare for, teach, and assess each page. Common
that each of the 7 topic areas, as well as the science student misconceptions are outlined in the beginning
practices, receives adequate attention. of each unit to help teachers understand what students
struggle with most.
AP Physics 1: Algebra-Based Course and Exam Description Instructional Approaches V.1 | 141
Return to Table of Contents
© 2021 College Board
Guided Inquiry in
AP Physics 1
The more active students are in their science students mirror how scientists analyze the natural
education, the more scientifically literate they will world. Inquiry requires identifying assumptions, using
become. Inquiry into authentic questions generated critical and logical thinking, and considering alternative
from student experiences should be one of the central explanations. Having students probe for answers to
strategies when teaching AP Physics 1. By posing scientific questions will lead to a deeper understanding
questions, planning investigations, and reviewing what of scientific concepts.
is already known in light of experimental evidence,
3.1: The student can pose The student The student The student The student
scientific questions. works with sharpens or clarifies selects from a set determines the
a question a question provided of given questions question.
developed by by the teacher. or can modify a
the teacher. given question.
4.1: The student can The student The student is given The student is told to The student
justify the selection of is given data data to analyze. collect and analyze can determine
the kind of data needed and told how to certain data. what constitutes
to answer a particular analyze it. evidence and can
scientific question. collect it.
5.1: The student can analyze [Science The student is given The student is The student can
data to identify patterns Practice 5.1 possible relationships directed toward independently
or relationships. has only three or patterns. patterns or examine data
levels of inquiry relationships. and form links
instruction.] to explanations.
6.1: The student can justify The student is The student is given The student is guided The student creates
claims with evidence. provided with possible ways to through the process an explanation
evidence to use evidence to of formulating after summarizing
support a claim. create explanations. explanations the evidence.
from evidence.
6.4: The student can make The student is The student is given The student is The student can
claims and predictions given steps and broad guidelines coached in the form reasonable and
about natural phenomena procedures to to use in the development logical arguments
based on scientific theories make claims sharpening of claims of claims to communicate
and models. and predictions. and predictions. and predictions. explanations based
on scientific theories
and models.
AP Physics 1: Algebra-Based Course and Exam Description Instructional Approaches V.1 | 142
Return to Table of Contents
© 2021 College Board
Different types of lessons, and therefore different homework assignment, and then work together as
types of inquiry, are used throughout AP Physics 1. a class to develop it further, making sure that the
There is a continuum from more student-centered question, variables, and safety are addressed.
types of inquiry to more teacher-centered types. §§ Try a goal-oriented task: Completely remove the
Understanding the different types of inquiry can help procedure, and prompt students with a question
teachers scaffold the types of labs and activities to that asks them to achieve something they want
better meet the needs of their students. to do. At this point, it’s best to choose a lab that
Below are four suggestions to make labs and activities students already understand conceptually and that
more student-centered and inquiry-based uses simple, familiar equipment.
§§ Let them do the thinking: Students choose what
§§ Start small: Take out the “data” or “results” section
they will investigate. Facilitate their thought
from traditional labs. If the procedure is thorough
process without telling them what to do. A pre-lab
and simple enough, students can read and design
brainstorming session in small groups is helpful
the data and results sections on their own.
when having students develop a question to
§§ Tackle the procedure: Eventually, teachers will want investigate. It is important to provide students with
students to design their own experiments, but they some guidelines at this step. For example, students
may need some practice first. Remove the step need to think about a question, a hypothesis, and
numbers and shuffle the steps. Have the students materials before beginning an open-ended lab.
work in pairs to put the steps into the correct order. Seeing and approving this in the lab groups helps
Next, try having them write a procedure as a pre-lab boost students’ confidence.
AP Physics 1: Algebra-Based Course and Exam Description Instructional Approaches V.1 | 143
Return to Table of Contents
© 2021 College Board
Instructional Strategies
The AP Physics 1 course framework outlines the concepts and science practices
students must master in order to be successful on the AP Exam. To address those
concepts and science practices effectively, teachers should incorporate a variety of
instructional approaches and best practices into their daily lessons and activities.
Teachers can help students develop the science practices by engaging them in
learning activities that allow them to apply their understanding of course concepts.
Teachers may consider the following strategies as they plan instruction. Please note
they are listed alphabetically and not by order of importance or instruction.
Ask the Expert Students are assigned as “experts” Provides Assign students “experts”
on problems they have mastered; opportunities for on conservation of linear
groups rotate through the expert students to share momentum questions.
stations to learn about problems their knowledge Students rotate through
they have not yet mastered. and learn from stations in groups,
one another. working with the station
expert to justify a set of
claims with corresponding
physical laws.
Bar Chart Bar chart tasks have histograms for Bar chart tasks This strategy can be used
one or more quantities. Frequently, help students make with conservation laws.
histograms are given before meaning by asking Have students define the
and after some physical process them to translate system and then create bar
with one bar left off. Students are between before charts for before and after
asked to complete the bar chart by and after some some event. For example,
supplying the value of the missing physical process. students create an energy
quantity. These are a new type of bar chart for a ball rolling
representation, requiring students down an incline. Students
to translate between whatever other identify the system and then
representation they are using and create one set of charts for
this one. Bar chart tasks are usually the top of the incline and a
quite productive in helping students separate set of charts for
make meaning. the bottom of the incline.
AP Physics 1: Algebra-Based Course and Exam Description Instructional Approaches V.1 | 144
Return to Table of Contents
© 2021 College Board
Strategy Definition Purpose Example
Changing These tasks require students to Students create As students learn about
Representations translate from one representation pictures, tables, energy conservation, ask
(e.g., an electric field diagram) to graphs, lists, them to move between
another (e.g., an equipotential curves equations, models, different representations.
or surfaces diagram). Students and/or verbal For example, for a given
often learn how to cope with one expressions to situation involving
representation without really learning interpret text or data. energy conservation,
the role and value of representations This helps organize students should be able
and their relationship to problem information using to create a sketch of the
solving. Getting them to go back multiple ways identified system, a set of
and forth between/among different to present data conservation of energy
representations for a concept forces and answer a equations, sets of energy
them to develop a more robust question or show a bar charts and graphs of
understanding of each representation. problem’s solution. potential energy, kinetic
Among the representations that energy, total energy or
will be employed at times are combinations of the above
mathematical relationships, so this representations.
task can serve as a bridge between
conceptual understanding and
traditional problem solving.
Concept-Oriented These tasks involve an actual Involving an actual In Unit 7, teachers can
Demonstration demonstration, but with the students demonstration, demonstrate the soup
doing as much of the description, students are can race, where different
prediction, and explanation as asked to predict soup cans with identical
possible. Demonstrations should be and explain. diameters reach the
ones where students feel comfortable bottom of an incline at
making predictions about what will different times because of
happen, yet will produce results they the contents of the can.
do not expect. Students will be challenged
to explain the outcome of
the race in terms of physical
laws and theories.
Conflicting Conflicting contentions tasks These tasks help This strategy is useful for
Contentions present students with two or three contrast statements helping students begin to
statements that disagree in some of students’ alternate understand how to write a
way. The students have to decide conceptions with full argument. By providing
which contention they agree with physically accepted the arguments and having
and explain why. These tasks are very statements. students identify good
useful for contrasting statements claims (and not-so-good
of students’ alternate conceptions claims) and good evidence
with physically accepted statements. and reasoning (and
This process is facilitated in these not-so-good evidence and
tasks because they can be phrased reasoning), teachers can
as “which statement do you agree help scaffold the instruction
with and why” rather than asking of good argumentation for
which statement is correct or true. their students.
These tasks complement “What if
Anything Is Wrong?” tasks.
AP Physics 1: Algebra-Based Course and Exam Description Instructional Approaches V.1 | 145
Return to Table of Contents
© 2021 College Board
Strategy Definition Purpose Example
Construct an Students use mathematical reasoning Helps develop This strategy can be used
Argument to present assumptions about the process with word problems that
mathematical situations, support of evaluating do not lend themselves to
conjectures with mathematically mathematical immediate application of
relevant and accurate data, and information, a formula or mathematical
provide a logical progression of developing process. Teachers can
ideals leading to a conclusion that reasoning skills, provide distance and velocity
makes sense. and enhancing graphs that represent a
communication motorist’s behavior through
skills in supporting several towns on a map and
conjectures ask students to construct
and conclusions. a mathematical argument
either in defense of or
against a police officer’s
charge of speeding, given a
known speed limit.
Create a Plan Students analyze the tasks in a Assists in breaking When scaffolding for how
problem and create a process for tasks into smaller to design an experiment,
completing the tasks by finding the parts and identifying assigning small groups
information needed, interpreting data, the steps needed to analyze the tasks
choosing how to solve a problem, to complete the necessary to design the
communicating the results, and entire task. experiment is a good first
verifying accuracy. step. Students identify the
steps needed to answer the
question by collecting and
analyzing data. Included
in this discussion is a plan
for what to do with the
collected data.
Debriefing Students discuss the understanding Helps clarify In order to discern the
of a concept to lead to a consensus misconceptions difference between average
on its meaning. and deepen velocity and instantaneous
understanding velocity, have students roll
of context. a ball down a simple ramp
and measure the distance
the ball travels over time
every second for 5 seconds.
Plotting position versus
time and sketching a curve
of best fit, students discuss
how they might determine
the average velocity of the
ball over the 5 seconds
and then the instantaneous
velocity of the ball at several
points. A discussion in
which students address the
distinction between the ball’s
average velocity between
two points and its velocity
at a single point helps in
clarifying the concept and
mathematical process.
AP Physics 1: Algebra-Based Course and Exam Description Instructional Approaches V.1 | 146
Return to Table of Contents
© 2021 College Board
Strategy Definition Purpose Example
Friends Without Students are given a free-response This can be a After scaffolding
Pens problem, quiz, or challenging problem. scaffolding tool argumentation, where
This task takes place in two rounds: if students are students have been working
The first round is the “friends without being introduced on identifying good (and
pens” round, where students are to a new type of not-so-good) claims,
grouped together and can discuss the assignment or a evidence, and reasoning,
question but are not allowed to write particularly difficult assigning a friends without
down anything. This round is timed. At or challenging pens task will help scaffold
the end of the time, students return AP-level question. one more step. Students
to their desks for the “pens without will identify good claims,
friends” round, where they tackle evidence, and reasoning
the assignment in the traditional, with their peers, and then
independent sense. they must return to their
desk, having discussed
the ideas, to create the
full argument.
AP Physics 1: Algebra-Based Course and Exam Description Instructional Approaches V.1 | 147
Return to Table of Contents
© 2021 College Board
Strategy Definition Purpose Example
Four-Square Students are given some sort of Re-expressing or In unit 4, students can
Problem Solving situation, perhaps one that came re-representing regularly and repeatedly
from a traditional, plug-and-chug data is a key skill for do four-square problem
problem. Have them divide a sheet student success solving with work and
of paper into four quadrants. In in AP Physics 1. energy questions. They can
each quadrant, the students are to Asking students to sketch graphs or free-body
put some representation of what is come up with four diagrams, write paragraphs,
going on in the problem. Possible representations and solve numerical and/or
representations include motion scaffolds the symbolic problems.
maps or graphs, any other kinds of practice needed
graphs, free-body diagrams, energy for them to get
bar graphs, momentum bar graphs, into the habit of
mathematical models (equations creating and using
with symbols), well-labeled diagrams, representations to
or written responses (two to three make claims and
strong, clear sentences). answer questions.
Graph and Students generate a graph (or Allows students As students learn about
Switch a sketch of a graph) to model a to practice momentum diagrams, have
certain function and then switch creating different them graph momentum
calculators (or papers) to review each representations versus time and force
other’s solutions. of functions as versus time and create
well as both giving a momentum diagram to
and receiving diagram a single situation.
feedback on each Students individually graph
other’s work. and explain how their
representations support
claims. They then share their
steps with a partner and
receive feedback on their
graphs, claims, evidence,
and reasoning.
AP Physics 1: Algebra-Based Course and Exam Description Instructional Approaches V.1 | 148
Return to Table of Contents
© 2021 College Board
Strategy Definition Purpose Example
Identify Subtasks Students break a problem into smaller Helps organize the Another scaffolding
pieces whose outcomes lead to a pieces of a complex technique: When first
solution. problem and reach a exposing students to
complete solution. AP-level questions that
involve several steps
of reasoning and logic,
additional questions can
be added to help guide
students to the final claim,
evidence, and reasoning.
For example, have students
sketch a free-body diagram,
discuss the system, and/or
draw energy bar charts.
After the first few units,
students should be able
to identify (first in groups
and then individually)
what the subtasks would
be (free-body diagram,
etc.) to start thinking
about the claim, evidence,
and reasoning.
AP Physics 1: Algebra-Based Course and Exam Description Instructional Approaches V.1 | 149
Return to Table of Contents
© 2021 College Board
Strategy Definition Purpose Example
Marking the Text Students highlight, underline, and/ Helps the student This strategy can be used
or annotate text to focus on key identify important with AP-level problems as
information to help understand the information in the well as problems from the
text or solve the problem. text and make notes text and sample laboratory
in the text about the procedures. Have students
concepts and read through the question,
interpretations of experimental design,
tasks required to or another student’s
reach a solution. experimental design and
underline the pronouns,
equipment, and key
information (i.e., the car
begins at rest) to identify
important information
and be able to ask
clarifying questions.
Meaningful, These tasks present the students Students are These calculations can take
Meaningless with an unreduced expression for a presented with many forms, but the most
Calculations calculation for a physical quantity for an unreduced useful are those where the
a physical situation. They must decide calculation for a “meaningless” calculations
whether the calculation is meaningful physical calculation illustrate common student
(i.e., it gives a value that tells us that involves a misconceptions. Students
something legitimate about the mathematical could be asked to write an
physical situation) or is meaningless relationship, and expression for the energy
(i.e., the expression is a totally students are asked of a system. Students then
inappropriate use of a relation). These if the calculation have to decide which of the
calculations should not be what we makes any sense. following expressions are
might call trivially meaningless, such meaningful (MgD, Mg/D,
as substituting a wrong numerical MD/g and 1/MgD).
value into the expression. These
items are best when the quantity Students could also be
calculated fits with students’ asked about a situation
alternative conceptions. where a cart with a fan is
released from rest and
moves across a flat tabletop
1 m long with negligible
friction. Have students find
the final speed of the cart by
measuring the time it took
to travel 1 m and dividing
the displacement of the cart
(1 m) by the time the cart
took to travel the 1 m. Ask
students if this a meaningful
calculation for this situation.
AP Physics 1: Algebra-Based Course and Exam Description Instructional Approaches V.1 | 150
Return to Table of Contents
© 2021 College Board
Strategy Definition Purpose Example
Model Questions Students answer items from released Provides rigorous Model questions can be
AP Physics Exams. practices and AP-released or AP-level
assesses students’ questions. They can be
ability to apply given as is, or scaffolded
multiple physical for students earlier in
practices on content the year to provide them
in either a multiple- with support.
choice or a free-
response question.
Note Taking Students create a record of Helps in Have students write down
information while reading a text or organizing ideas verbal descriptions of the
listening to a speaker. and processing steps needed to solve a
information. problem so that a record
of the processes can be
referred to at a later point
in time.
Predict and Predict and explain tasks describe Stimulates thinking When a ballistic pendulum
Explain a physical situation that is set up by asking students is set up, ask students what
at a point where some event is to make, check, and will happen to the maximum
about to occur. Students predict correct predictions swing height when the mass
what will happen in the situation based on evidence of the dart is increased or
and explain why they think that will from the outcome. decreased? What would
occur. These tasks must involve happen if the dart were to
situations with which the students bounce instead of stick into
are familiar or have sufficient the block? What if the dart
background information to enable passed through the block?
them to understand the situation.
This is important because otherwise
they usually do not feel comfortable
enough to attempt to answer.
Qualitative These tasks can take a variety Students are Ask students what would
Reasoning of forms, with their common presented with a happen to the angular
denominator being qualitative physical situation momentum of an object
analysis. Frequently, students are and asked to apply in orbit around the Earth
presented with an initial and a final a principle to if the radius of orbit were
situation and asked how some qualitatively reason increased, if the speed of
quantity, or aspect, will change. what will happen. orbit were decreased, or
Qualitative comparisons (e.g., the These questions if the mass of the Earth
quantity increases, decreases, are commonly were changed. Additional
or stays the same) are often the found in other questions could include,
appropriate answer. Qualitative multiple-choice “What happens to the
reasoning tasks can frequently question subtypes. energy of the system as the
contain elements found in some of physical properties above
the other task formats (e.g., different are changed?”
qualitative representations and a
prediction or an explanation).
AP Physics 1: Algebra-Based Course and Exam Description Instructional Approaches V.1 | 151
Return to Table of Contents
© 2021 College Board
Strategy Definition Purpose Example
Quickwrite Students write for a short, Helps generate ideas To help synthesize concepts
specific amount of time about a in a short amount after having learned
designated topic. of time. about the conservation of
mechanical energy, have
students list as many ways
as possible to change the
total mechanical energy
of a system and how each
change affects the total
mechanical energy.
Ranking Ranking tasks present students These tasks require Given six different arrows
with a set of variations, sometimes students to engage launched from the ground
three or four but usually six to eight, in a comparison with different speeds at
on a basic physical situation. The reasoning process different angles, have
variations differ in the values (numeric that they seldom students rank the arrows
or symbolic) for the variables involved have opportunities on the basis of the highest
but also frequently include variables to do in traditional acceleration at the top, the
that are not important to the task. problem solving. longest time in the air, and
The students’ task is to rank the the largest velocity at the top.
variations on the basis of a specified
physical quantity. Student must also
explain the reasoning for their ranking
scheme and rate their confidence in
their ranking.
Sharing and Students communicate with Gives students Group students to review
Responding another person or a small group of the opportunity to individual work (graphs,
peers who respond to a proposed discuss their work derivations, problem
problem solution. with peers, make solutions, experimental
suggestions to designs, etc.). Have the
improve the work groups make any necessary
of others, and/or corrections and build a
receive appropriate single, complete solution
and relevant together.
feedback on their
own work.
Simplify the Students use friendlier numbers or Provides insight into Have students use the
Problem functions to help solve a problem. the problem or the analogy of one-dimensional
strategies needed to motion when initially
solve the problem. analyzing rotational
kinematics.
AP Physics 1: Algebra-Based Course and Exam Description Instructional Approaches V.1 | 152
Return to Table of Contents
© 2021 College Board
Strategy Definition Purpose Example
Troubleshooting Troubleshooting tasks are variations Allows students to Give students a derivation
on the “What if Anything Is Wrong?” troubleshoot errors or problem solution and ask
tasks. Students are explicitly told that and misconceptions them to find the incorrect
there is an error in the given situation. by focusing on step(s). Have them identify
Their job is to determine what the problems that may and explain the mistake
error is and explain how to correct arise when they do or misunderstanding that
it. These tasks can often produce the same procedures led to the error. This can
interesting insights into students’ themselves. also be done with bar
thinking, because they will, at times, charts, diagrams, and
identify some correct aspect of the other representations.
situation as erroneous. This helps
develop additional items.
Write and Switch Like graph and switch, with writing. Allows students As students learn about
Students make observations or to practice writing creating an argument,
collect data or make a claim and then and both give and they can draft an initial
switch papers. receive feedback on argument themselves; share
each other’s work. their claim, evidence, and
reasoning with a partner;
and receive feedback on
their argument.
AP Physics 1: Algebra-Based Course and Exam Description Instructional Approaches V.1 | 153
Return to Table of Contents
© 2021 College Board
Strategy Definition Purpose Example
Working This task reverses the order of the Provides another Students are given
Backward problem steps. For example, the given way to check an equation such as
information could be an equation with possible answers 4m = 6 m − 9 m2 and are
specific values for all, or all but one, for accuracy. s s
of the variables. The students then asked to create another
have to construct a physical situation representation from this
for which the given equation would equation, such as a written
apply. Such working backward tasks scenario that this equation
require students to take numerical could represent, a position
values, including units, and translate versus time graph, a
them into physical variables. Working velocity versus time graph,
backward problems also require a motion map, etc.
students to reason about these
situations in an unusual way and often
allow for more than one solution.
AP Physics 1: Algebra-Based Course and Exam Description Instructional Approaches V.1 | 154
Return to Table of Contents
© 2021 College Board
Developing the
Science Practices
Throughout the course, students will develop science practices that are
fundamental to the discipline of physics. Since these practices represent the
complex skills that adept physicists demonstrate, students will benefit from multiple
opportunities to develop them in a scaffolded manner. Through the use of guided
questioning, discussion techniques, and other instructional strategies, teachers
can help their students practice applying these science practices in new contexts,
providing an important foundation for their college and career readiness.
1.1 The student can §§ What kind of model or Have students divide their §§ Four-Square
create representations representation would paper into four quarters. Problem Solving
be appropriate for In each quarter of the §§ “What if Anything
and models of natural or
this physical system? paper, students create a Is Wrong?”
man-made phenomena representation of the physical
§§ What physical §§ Graph and Switch
and systems in characteristics situation. Representations can
the domain. include equations, sentences §§ Changing
can be modeled or
(or paragraphs), bar charts, Representations
represented for this
physical situation? circuit diagrams, or sketches of
physical situations.
AP Physics 1: Algebra-Based Course and Exam Description Instructional Approaches V.1 | 155
Return to Table of Contents
© 2021 College Board
Instructional Notes or Instructional
Science Practice Key Questions Sample Activities Strategies
1.2 The student can §§ What does the Have students describe the §§ “What if Anything Is
describe representations representation show? physical features and meaning Wrong?”
of figures and representations, §§ Graph and Switch
and models of natural or
including figures and
man-made phenomena and §§ Discussion Groups
representations from the textbook
systems in the domain. or other reference sources.
1.3 The student can §§ What assumptions Have students in groups create §§ Graph and Switch
refine representations and are inherent in the a representation for a certain §§ “What if Anything Is
representation physical situation. The groups Wrong?”
models of natural or man-
or model? then switch papers and discuss
made phenomena and modifications that can be
§§ How can these
systems in the domain. assumptions be made to the representations or
modified in the models based on assumptions
representation that may have been made
or model? or could be made about the
physical situation.
§§ What would the
representation or
model look like if
these assumptions
were modified?
1.4 The student can §§ What does the Have students analyze slopes, §§ “What if Anything Is
use representations representation show? areas under curves, and y Wrong?”
§§ What features of and x intercepts to help them §§ Changing
and models to analyze
the representation solve problems. Students Representations
situations or solve should be able to analyze
provide information §§ Bar Chart
problems qualitatively relevant to the situations using graphs/models/
and quantitatively. question or problem? representations as easily as they
can with numbers and equations.
1.5 The student can §§ What characteristic Have students divide their §§ “What if Anything Is
re-express key elements or physical quantity paper into four quarters Wrong?”
of the situation does and provide four different §§ Changing
of natural phenomena
each representation representations for a Representations
across multiple illustrate? given physical situation.
representations in §§ Four-Square
§§ How do the Representations can include
Problem Solving
the domain. representations an equation, a written sentence
show consistency? (or paragraph), a graph, a
bar chart, or a sketch of the
physical scenario.
AP Physics 1: Algebra-Based Course and Exam Description Instructional Approaches V.1 | 156
Return to Table of Contents
© 2021 College Board
Science Practice 2: Mathematical Routines
The student can use mathematics appropriately. mathematical representation, instead of first choosing
a formula whose variables seem to match the givens
Physicists commonly use mathematical
in the problem. Students should also be able to
representations to describe and explain phenomena,
work with the algebraic form of the equation before
as well as to solve problems. When students work with
substituting values. Students should be able to
these representations, we want them to understand
evaluate the equation(s) and the answer in terms of
the connections between the mathematical
units and limiting case analysis. Students should be
description, the physical phenomena, and the concepts
able to translate between functional relationships in
represented in the mathematical descriptions. When
equations (proportionalities, inverse proportionalities,
using equations or mathematical representations,
etc.) and cause-and-effect relationships in the physical
students need to be able to justify why using a
world. Students should be able to evaluate a numerical
particular equation to analyze a particular situation is
result in terms of whether it makes sense. In many
useful and to be aware of the conditions under which
physical situations, simple mathematical routines may
the equations/mathematical representations can be
be needed to arrive at a result even though they are
used. Students tend to rely too much on mathematical
not the focus of a learning objective.
representations. When solving a problem, they
need to be able to describe the problem situation The following table provides examples of questions and
in multiple ways, including pictorial representations, instructional strategies for implementing mathematical
force diagrams etc., and then choose an appropriate routines into the course:
2.1 The student can §§ What quantities are given? Have students work backward §§ Working
justify the selection §§ What quantity is needed to from a given mathematical Backward
answer the question? routine to a physical situation. §§ Simplify the
of a mathematical
For example, students can Problem
routine to §§ What relationship(s) link the
be given an equation such as
needed quantities with the §§ Ask the Expert
solve problems.
given quantities?
4m = 6 m − 9 m2 and then
s s
be asked to create another
representation from this
equation, such as a written
scenario that this equation could
represent, a position versus time
graph, a velocity versus time
graph, a motion map, etc.
AP Physics 1: Algebra-Based Course and Exam Description Instructional Approaches V.1 | 157
Return to Table of Contents
© 2021 College Board
Instructional Notes or Instructional
Science Practice Key Questions Sample Activities Strategies
2.2 The student can §§ What laws, definitions, or Have students perform a §§ Model
apply mathematical mathematical relationships exist task where the calculations Questions
that relate to the given problem? (plugging and chugging) are §§ Discussion
routines to quantities
§§ What are the rules, assumptions, already done—a task that will Groups
that describe natural force the students to focus on
or limitations surrounding the §§ Meaningful,
phenomena. use of the chosen law, definition, making important distinctions
Meaningless
or relationship? that physicists consider
Calculations
critical. For example, have
§§ Did the calculation begin with
students determine whether
an equation or a fundamental
a specified calculation is
physics relationship, law,
meaningful or meaningless—
or definition?
this is entirely different from
§§ Are the steps clearly written out plugging numerical values
and annotated? into an equation and turning
§§ Are any steps skipped? the crank.
§§ Is the unknown quantity clearly
Meaningful, meaningless
labeled as the final answer,
calculation tasks are another
complete with units?
tool to get students to process
information about a concept or
principle in a different way.
2.3 The student can §§ How can the mathematical Have students practice §§ Meaningful-
estimate numerically routine be simplified to give estimating numerical Meaningless
an estimated or order-of- quantities. For example, have Calculations
quantities that
magnitude calculation? students practice determining §§ Simplify the
describe natural the approximate center
§§ How can this estimated value be Problem
phenomena. used as a guide when calculating of mass for a system of
an unknown value? discrete objects.
AP Physics 1: Algebra-Based Course and Exam Description Instructional Approaches V.1 | 158
Return to Table of Contents
© 2021 College Board
Science Practice 3: Scientific Questioning
The student can engage in scientific questioning to and cognitive goal. Students need to be guided away
extend thinking or to guide investigations within the from asking “fuzzy” questions about queries that
context of the AP course. are measurable and testable. A first step in refining
questions might be to guide students to consider all the
Research scientists pose and answer meaningful
ways one might measure relevant physical quantities,
questions. Students may easily miss this point since,
leading to further discussions about how one would
depending on how a science class is taught, it may
evaluate questions by designing and carrying out
seem that science is about compiling and passing
experiments and then evaluating data and findings.
down a large body of known facts (e.g., the acceleration
of a free-falling object is 9.8 m/s2). Helping students The following table provides examples of questions
learn how to pose, refine, and evaluate scientific and instructional strategies for implementing scientific
questions is an important but difficult instructional questioning into the course:
3.1 The student §§ What does it mean Have students practice posing §§ Desktop
can pose scientific for a question to scientific questions by giving Experiment Tasks
be “scientific?” them opportunities to discuss §§ Write and Switch
questions.
§§ How can questions what is scientifically measurable
be modified to make and determinable with certain
them testable? laboratory equipment.
3.2 The student §§ How can scientific Have students practice engaging §§ Desktop
can refine scientific questions be modified in scientific questioning. For Experiment Tasks
to make them testable? example, have students start §§ Write and Switch
questions.
§§ How can a scientific by asking questions about the
§§ Predict and Explain
question be refined relationships between linear
concerning a proposed momentum and kinetic energy
incorrect relationship before and after a collision. As a
between variables? first step, students might consider
in what ways one can measure
physical quantities relevant to the
collision, leading to a discussion
of mass, velocity, momentum,
and kinetic energy. Follow-up
discussions can lead to how one
goes about evaluating questions
such as, “Does the conservation
of momentum and/or kinetic
energy depend on the type of
collision?” by designing and
carrying out experiments and
then evaluating data and findings.
AP Physics 1: Algebra-Based Course and Exam Description Instructional Approaches V.1 | 159
Return to Table of Contents
© 2021 College Board
Science Practice 4: Experimental Methods
The student can plan and implement data- a numerical answer to a question are best thought of
collection strategies in relation to a particular as an interval, not a single number. This interval, the
scientific question. experimental uncertainty, is due to a combination of
uncertainty in the instruments used and the process
Scientific questions can range in scope, from broad
of taking the measurement. Although detailed error
to narrow, as well as in specificity, from determining
analysis is not necessary to convey this pivotal idea, it is
influencing factors and/or causes to determining
important that students make some reasoned estimate
mechanisms. The question posed will determine the
of the interval within which they know the value of a
type of data to be collected and will influence the
measured data point and can express their results in a
plan for collecting data. Designing and improving
way that makes this clear.
experimental designs and/or data-collection strategies
is a learned skill. Class discussion can reveal issues The following table provides examples of questions and
of measurement uncertainty and assumptions in instructional strategies for implementing data-collection
data collection. Students need to understand that resources into the course:
the results of collecting and using data to determine
4.1 The student §§ What data is necessary Have students practice justifying §§ Discussion
can justify the to answer the scientific the selection of the kind of data Groups
question? needed to answer a particular §§ Create a Plan
selection of the kind
§§ What physical law, scientific question.
of data needed to §§ Write and Switch
equation, or relationship
answer a particular For example, have students
links the scientific question
scientific question. design an experiment and
with the collected data?
analyze data to determine
the relationship between the
change in angular momentum
of a system and the product
of the average torque applied
to the system and the time
interval during which the torque
is exerted.
4.2 The student can §§ What information will be Have students practice §§ Create a Plan
design a plan for needed to answer the designing plans for collecting §§ Troubleshooting
scientific question? data to answer scientific
collecting data to §§ Desktop
§§ What equipment is needed to questions. Laboratory design Experiment Tasks
answer a particular procedures do not always have
collect the necessary data?
scientific question. to be carried out.
§§ How will each piece of
equipment be used to For example, have students
collect the necessary data? design an experiment and
§§ What will be done with analyze graphical data where the
the data (data analysis) area under a velocity versus time
to answer the scientific graph is needed to determine the
question? displacement of an object.
AP Physics 1: Algebra-Based Course and Exam Description Instructional Approaches V.1 | 160
Return to Table of Contents
© 2021 College Board
Instructional Notes or Instructional
Science Practice Key Questions Sample Activities Strategies
4.3 The student §§ What information will be Have students follow through §§ Write and Switch
can collect data to needed to answer the with plans they have designed §§ Desktop
scientific question? to collect data to answer a Experiment Tasks
answer a particular
§§ What equipment is scientific question. Students
scientific question. can also be given a procedure
needed to collect the
necessary data? to follow and can practice
collecting careful data from
§§ How will each piece of
a teacher’s or classmate’s
equipment be used to
written instructions.
collect the necessary data?
§§ What will be done with
the data (data analysis)
to answer the scientific
question?
4.4 The student can §§ Can the data set given or Expose students to data that §§ Troubleshooting
evaluate sources of collected be trusted? might have anomalies or might §§ Desktop
§§ Could there be anomalies not be accurate. Students Experiment Tasks
data to answer a
in the data that need to can discuss the possible
particular scientific reasons why a particular data
be resolved?
question. set is or is not reliable. For
example, students should be
able to analyze data to verify
conservation of momentum
with and without an external
frictional force.
AP Physics 1: Algebra-Based Course and Exam Description Instructional Approaches V.1 | 161
Return to Table of Contents
© 2021 College Board
Science Practice 5: Data Analysis
The student can perform data analysis and quantity and then check for consistency across the
evaluation of evidence. two measurements, comparing two results by writing
them both as intervals and not as single, absolute
Students often think that to make a graph, they need
numbers. Finally, students should be able to revise
to connect the data points, or that the best-fit function
their reasoning based on the new data, which for some
is always linear. Thus, it is important that they can
may appear anomalous. The analysis, interpretation,
construct a best-fit curve even for data that do not
and application of quantitative information are vital
fit a linear relationship. Students should be able to
skills for students in AP Physics 1. Analysis skills can
represent data points as intervals whose size depends
be taught using any type of data, but students will be
on the experimental uncertainty. After students find a
more invested in the data analysis if it is data they have
pattern in the data, they need to ask why this pattern
collected through their own investigations. Teachers
is present and try to explain it using the knowledge
are encouraged to provide opportunities for students
that they have. When dealing with a new phenomenon,
to analyze data, draw conclusions, and apply their
they should be able to devise a testable explanation
knowledge to the enduring understandings and learning
of the pattern, if possible. It is important that students
objectives in the course.
understand that instruments do not produce exact
measurements and learn what steps they can take The following table provides examples of questions and
to decrease uncertainty. Students should be able to instructional strategies for implementing data analysis
design a second experiment to determine the same resources into the course:
5.1 The student §§ How should the data be graphed Have students practice §§ Friends
can analyze data so that the best-fit curve shows analyzing data to find patterns Without Pens
a relationship? and relationships. For example, §§ Write and
to identify patterns
§§ How can data intervals have students analyze data Switch
or relationships. (or a visual representation)
be used to show §§ Graph and
experimental uncertainty? describing the motion of
Switch
an object and express the
§§ What do the data or graph show? §§ Predict and
results of the analysis using
§§ What trends and patterns can narrative, mathematical, and Explain
you identify from the data? graphical representations.
§§ Why is the pattern present in
the data? What does the pattern
show about the relationship
between quantities?
5.2 The student can §§ What changes can be Have students practice §§ Desktop
refine observations made to observations and refining observations and Experiment
measurements to refine measurements. For example, Tasks
and measurements
the data? have students perform data §§ Write and
based on analysis, evaluate evidence
§§ How can a second experiment Switch
data analysis. be designed to answer the same compared to the prediction,
§§ Graph and
scientific question? explain any discrepancy,
Switch
and, if necessary, revise the
§§ What steps can be taken to
relationship among variables
decrease the uncertainty in the
responsible for the period of a
measurements and data?
pendulum.
AP Physics 1: Algebra-Based Course and Exam Description Instructional Approaches V.1 | 162
Return to Table of Contents
© 2021 College Board
Instructional Notes or Instructional
Science Practice Key Questions Sample Activities Strategies
5.3 The student §§ How does the presented Have students refine and §§ Sharing and
can evaluate the evidence provide support for analyze a scientific question Responding
the claim or scientific question? for an experiment relating the §§ Conflicting
evidence provided by
§§ Does the data set present net work on an object to the Contentions
data sets in relation forces exerted on that object
clear and complete evidence
to a particular in relation to the scientific over a distance.
scientific question. question, or is the data flawed?
§§ If the data is flawed, what
new data or procedure
should be completed to
obtain data in relation to the
scientific question?
AP Physics 1: Algebra-Based Course and Exam Description Instructional Approaches V.1 | 163
Return to Table of Contents
© 2021 College Board
Science Practice 6: Argumentation
The student can work with scientific explanations make predictions. A prediction states the expected
and theories. outcome of a particular experimental design based
on an explanation or a claim under scrutiny. Physicists
A scientific explanation, accounting for an observed
examine data and evidence to develop claims about
phenomenon, needs to be experimentally testable.
physical phenomena. As they articulate their claims,
One should be able to use it to make predictions about
physicists use reasoning processes that rely on
new phenomena. A theory uses a unified approach
their awareness of different types of relationships,
to account for a large set of phenomena and gives
connections, and patterns within the data and evidence.
accounts that are consistent with multiple experimental
They then formulate a claim and develop an argument
outcomes within the range of applicability of the theory.
that explains how the claim is supported by the
Examples of theories in physics include the kinetic
available evidence. AP Physics 1 teachers should help
molecular theory, quantum theory, and atomic theory.
students learn how to create persuasive and meaningful
Students should understand the difference between
arguments by improving their proficiency with each of
explanations and theories.
these practices.
Students should be prepared to offer evidence,
The following table provides examples of strategies for
construct reasoned arguments for their claim from
implementing argumentation resources into the course:
the evidence, and use the claim or explanation to
6.1 The student §§ What is evidence, Have students identify and explain §§ Conflicting
can justify claims and how does it differ the evidence that supports their Contentions
from reasoning? claim, with an emphasis on how
with evidence.
the evidence supports the claim.
6.2 The student §§ What possible claims Have students construct §§ Conflicting
can construct could you make based an explanation of physical Contentions
on the question and phenomena based on evidence. §§ Concept-Oriented
explanations
the evidence? For example, students can Demonstration
of phenomena construct an explanation of the
§§ What is your purpose §§ Discussion Groups
based on evidence (to define, show causality, inverse square dependence of
produced through compare, or explain the gravitational field surrounding
scientific practices. a process)? a spherically symmetric,
massive object.
§§ What evidence supports
your claim?
§§ How does the evidence
support your explanation?
AP Physics 1: Algebra-Based Course and Exam Description Instructional Approaches V.1 | 164
Return to Table of Contents
© 2021 College Board
Instructional Notes or Instructional
Science Practice Key Questions Sample Activities Strategies
6.4 The student can §§ What reasoning (physical Have students make claims about §§ Predict and
make claims and laws or theories) supports a physical situation that is set up Explain
your claim? at a point where some event is §§ Discussion Groups
predictions about
§§ How does the reasoning about to occur. Students have to
natural phenomena §§ Conflicting
support your claim? predict what will happen in the
based on scientific Contentions
situation and explain why they
§§ How does the evidence
theories and models. think that will occur. For example,
support your claim?
students can make claims and
Use transitions such as
predictions about the internal
because or therefore.
energy of systems.
AP Physics 1: Algebra-Based Course and Exam Description Instructional Approaches V.1 | 165
Return to Table of Contents
© 2021 College Board
Science Practice 7: Making Connections
The student is able to connect and relate in which major ideas apply facilitates transfer; this allows
knowledge across various scales, concepts, and students to bundle knowledge and memory together with
representations in and across domains. the multiple contexts to which it applies. Students should
also be able to recognize seemingly appropriate contexts
Students should have the opportunity to transfer their
to which major concepts and ideas do not apply.
learning across disciplinary boundaries so that they are
able to link, synthesize, and apply the ideas they learn The following table provides examples of questions
across the sciences and mathematics. Research on how and instructional strategies for making connections
people learn indicates that providing multiple contexts throughout the course:
7.1 The student can §§ What models and/or Have students practice §§ Meaningful,
connect phenomena representations can help connecting phenomena Meaningless
connect these phenomena across spatial and temporal Calculations
and models
with other phenomena? scales. Problem solving in §§ Model Questions
across spatial and isolation of one unit will not
§§ What important features §§ “What if Anything Is
temporal scales. of the models and/or prepare students for the
Wrong?”
representations connect AP Physics 1 Exam.
the phenomena across
Have students connect
spatial and temporal scales?
representations between
topics and big ideas.
7.2 The student §§ What big ideas can link Have students practice §§ Meaningful,
can connect these phenomena with connecting phenomena Meaningless
other phenomena? across domains and making Calculations
concepts in and
§§ How can the ideas used to generalizations across §§ Model Questions
across domain(s) enduring understandings and
explain this phenomenon §§ “What if Anything Is
to generalize be generalized to big ideas. For example, have
Wrong?”
or extrapolate in extrapolate across students apply conservation
and/or across enduring understandings? concepts for energy, charge,
enduring and linear momentum to
§§ How can the ideas used to
everyday situations.
understandings explain this phenomenon be
and/or big ideas. generalized to extrapolate
across big ideas?
AP Physics 1: Algebra-Based Course and Exam Description Instructional Approaches V.1 | 166
Return to Table of Contents
© 2021 College Board
AP PHYSICS 1
Exam
Information
Exam Overview
Number of
Section Question Type Questions Weighting Timing
Single-select multiple-choice 45
IA
questions (discrete or in sets)
50% 90 minutes
Multiple-select multiple-choice 5
IB
items (all discrete)
The exam assesses content from each of five big ideas for the course:
AP Physics 1: Algebra-Based Course and Exam Description Exam Information V.1 | 169
Return to Table of Contents
© 2021 College Board
The exam also assesses each of the 10 units of instruction with the following exam
weightings on the multiple-choice section of the AP Exam:
AP Physics 1: Algebra-Based Course and Exam Description Exam Information V.1 | 170
Return to Table of Contents
© 2021 College Board
How Student Learning Is
Assessed on the AP Exam
Section I: Multiple-Choice
Science Practices 1, 2, 4, 5, 6, and 7 are all assessed in in the multiple-choice section with the following weighting
(Science Practice 3 will not be assessed in the multiple-choice section):
AP Physics 1: Algebra-Based Course and Exam Description Exam Information V.1 | 171
Return to Table of Contents
© 2021 College Board
Section II: Free-Response
Science Practices 1, 2, 4, 5, 6, and 7 are all assessed in in the free-response section with the following weighting
(Science Practice 3 will not be assessed in the free-response section):
The AP Physics 1 free-response section includes five free-response questions: two 12-point questions and three
7-point questions. Every exam includes one experimental design question, one quantitative/qualitative translation
question, one paragraph short answer question, and two additional short answer questions. These questions may
appear in any order on the AP Exam.
AP Physics 1: Algebra-Based Course and Exam Description Exam Information V.1 | 172
Return to Table of Contents
© 2021 College Board
Task Verbs Used in
Free-Response
Questions
The following task verbs are commonly used in the free-response questions.
Calculate: Perform mathematical steps to arrive at a final answer, including
algebraic expressions, properly substituted numbers, and correct labeling of
units and significant figures. Also phrased as “What is?”
Compare: Provide a description or explanation of similarities and/or differences.
Derive: Perform a series of mathematical steps using equations or laws to
arrive at a final answer.
Describe: Provide the relevant characteristics of a specified topic.
Determine: Make a decision or arrive at a conclusion after reasoning,
observation, or applying mathematical routines (calculations).
Evaluate: Roughly calculate numerical quantities, values (greater than, equal
to, less than), or signs (negative, positive) of quantities based on experimental
evidence or provided data. When making estimations, showing steps in
calculations are not required.
Explain: Provide information about how or why a relationship, pattern, position,
situation, or outcome occurs, using evidence and/or reasoning to support
or qualify a claim. Explain “how” typically requires analyzing the relationship,
process, pattern, position, situation, or outcome; whereas, explain “why”
typically requires analysis of motivations or reasons for the relationship,
process, pattern, position, situation, or outcome.
Justify: Provide evidence to support, qualify, or defend a claim, and/or provide
reasoning to explain how that evidence supports or qualifies the claim.
Label: Provide labels indicating unit, scale, and/or components in a diagram,
graph, model, or representation.
Plot: Draw data points in a graph using a given scale or indicating the scale and
units, demonstrating consistency between different types of representations.
Sketch/Draw: Create a diagram, graph, representation, or model that illustrates
or explains relationships or phenomena, demonstrating consistency between
different types of representations. Labels may or may not be required.
State/Indicate/Circle: Indicate or provide information about a specified topic,
without elaboration or explanation. Also phrased as “What…?” or ”Would…?”
interrogatory questions.
Verify: Confirm that the conditions of a scientific definition, law, theorem, or test are
met in order to explain why it applies in a given situation. Also, use empirical data,
observations, tests, or experiments to prove, confirm, and/or justify a hypothesis.
AP Physics 1: Algebra-Based Course and Exam Description Exam Information V.1 | 173
Return to Table of Contents
© 2021 College Board
Sample Exam
Questions
The sample exam questions that follow illustrate the relationship between the course
framework and AP Physics 1 Exam and serve as examples of the types of questions
that appear on the exam. After the sample questions, teachers will find a table that
shows which science practice(s), learning objective(s), and unit each question relates
to. The table also provides the answers to the multiple-choice questions.
1. Block Y with mass mY falls onto and sticks to block X , which is attached to a
vertical spring, as shown in Figure 1. A short time later, as shown in Figure 2,
the blocks are momentarily at rest. At that moment, block Y exerts a force of
magnitude Fdown on block X , and block X exerts a force of magnitude Fup on
block Y . Which of the following correctly relates Fup, Fdown, and mY g at the instant
shown in Figure 2?
(A)
Fup Fdown mY g
F
(B) up m gF
Y down
AP Physics 1: Algebra-Based Course and Exam Description Exam Information V.1 | 174
Return to Table of Contents
© 2021 College Board
2. A block is released from rest and slides down a ramp. The surface of the ramp
has three rough sections where the friction between the surface and the block
is not negligible, as shown by the shaded regions above. Measuring which
of the following will allow for the best estimate of the block’s instantaneous
acceleration when the block is at the midpoint of the ramp?
(A) The total distance traveled by the block and the total elapsed time
(B) The final speed of the block and the total elapsed time
(C) The distance between points just before and just after the midpoint and the
time it takes the block to travel between them
(D) The speed of the block at points just before and just after the midpoint and
the time it takes the block to travel between them
AP Physics 1: Algebra-Based Course and Exam Description Exam Information V.1 | 175
Return to Table of Contents
© 2021 College Board
Questions 4–6 refer to the following material.
The figures above show a small block of mass 0.20 kg on a track in the shape of a
circular arc. The block is released from rest at a height H above the floor, as shown in
Figure 1. The block slides along the track with negligible friction and leaves it at a height
of 0.40 m above the floor and a speed of 3.0 m/s at a 30 angle, as shown in Figure 2.
5. The magnitude of the gravitational force exerted on the block is Fg, and the
magnitude of the normal force exerted by the track on the block is Fn. Which of
the following correctly compares the magnitudes of these two forces when the
block is at the lowest point on the track?
(A) Fn Fg
(B) Fn Fg
(C) Fn Fg
(D) The magnitudes cannot be compared without knowing the radius of the arc
of the track.
6. After the block leaves the track, what is the block’s speed when it reaches the
highest point of its motion?
(A) 0
(B) 1.5 m/s
(C) 2.6 m/s
(D) 3.0 m/s
AP Physics 1: Algebra-Based Course and Exam Description Exam Information V.1 | 176
Return to Table of Contents
© 2021 College Board
7. Objects X and Y are constrained to move along a straight line. The graphs above
show the net force exerted along that line on each of the objects as functions of
time. Which of the following correctly ranks the change in momentum p of the
objects?
(A) pX pY
(B) pX
pY
(C) pX pY
(D) The ranking cannot be determined without knowing the masses of the objects.
8. A person exerts an upward force on a box, as shown above. The box may be
moving upward, downward, or not at all while the person exerts the upward
force. For which of the following motions of the box is the work done by the
person on the box correctly indicated?
Motion of Box Work Done by Person on Box
(A) No motion Positive
(B) Upward with decreasing speed Negative
(C) Downward with constant speed Zero
(D) Downward with increasing speed Negative
AP Physics 1: Algebra-Based Course and Exam Description Exam Information V.1 | 177
Return to Table of Contents
© 2021 College Board
9. Two satellites are in circular orbits around Earth. Satellite 1 has mass m0 and an
orbital radius of 2RE, where RE is the radius of Earth. Satellite 2 has mass 2m0
and an orbital radius of 3RE. Which of the following correctly compares the
magnitude F of the force exerted by Earth on each satellite and the speed v of
each satellite?
Force Speed
(A) F1 > F2 v1 > v2
(B) F1 > F2 v2 > v1
(C) F2 > F1 v1 > v2
(D) F2 > F1 v2 > v1
10. In the setup shown above, a student uses motion detector 1 to measure the
speed vi of a cart with mass m before it collides with and sticks to a stationary
cart with mass M. Motion detector 2 measures the speed v f of the carts after the
collision. The student repeats the experiment several times using different values
of vi and creates a graph of v f as a function of vi. The slope of this graph is most
nearly equal to
m
(A)
M
m
(B)
M m
M m
(C)
M m
m
(D)
M m
AP Physics 1: Algebra-Based Course and Exam Description Exam Information V.1 | 178
Return to Table of Contents
© 2021 College Board
11. Cylindrical disk A is rotating freely about an axis when an identical disk B that is
not rotating is dropped directly on top of disk A. If the two disks stick together,
how does the total angular momentum and total kinetic energy of the two-disk
system after the disks are stuck together compare to that of the system before
disk B was dropped?
Total Angular Momentum Total Kinetic Energy
(A) Remains the same Is one-half its original value
(B) Remains the same Is one-fourth its original value
(C) Is one-half its original value Is one-half its original value
(D) Is one-half its original value Is one-fourth its original value
12. Which of the following statements about a satellite in an elliptical orbit around
Earth are correct? Select two answers.
(A) The satellite’s kinetic energy is constant throughout the orbit.
(B) The satellite’s angular momentum about the center of mass of the satellite-
Earth system is constant throughout the orbit.
(C) The magnitude of the satellite’s linear momentum is constant throughout
the orbit.
(D) The gravitational potential energy of the Earth-satellite system is greatest at
the satellite’s farthest point from Earth.
AP Physics 1: Algebra-Based Course and Exam Description Exam Information V.1 | 179
Return to Table of Contents
© 2021 College Board
Section II: Free-Response Questions
The following are examples of the kinds of free-response questions found on the exam.
Note that on the actual AP Exam, there will be one experimental design question, one
quantitative/qualitative translation question, one paragraph argument short answer
question, and two additional short answer questions.
AP Physics 1: Algebra-Based Course and Exam Description Exam Information V.1 | 180
Return to Table of Contents
© 2021 College Board
Instead of moving in a horizontal circle, the sphere now moves in a vertical
plane so that it is a simple pendulum, as shown above. The maximum angle
max that the string makes from the vertical can be assumed to be small.
The graph below shows data for the square of the pendulum period T as a
function of string length L.
(C) On the graph above, draw a best-fit line for the data. Then use the line to
calculate a numerical value for the gravitational acceleration g .
AP Physics 1: Algebra-Based Course and Exam Description Exam Information V.1 | 181
Return to Table of Contents
© 2021 College Board
(D) The graph above shows the angle from the vertical as a function of time
for the pendulum. On the axes below, sketch a graph of the gravitational
potential energy of the sphere-Earth system for the same time interval.
Take the zero of potential energy to be when the potential energy has its a
minimum value.
(E) As the sphere swings back and forth, it must also rotate a small amount
during each swing. The figures below indicate the direction that the sphere
rotates as it is swinging in each direction.
AP Physics 1: Algebra-Based Course and Exam Description Exam Information V.1 | 182
Return to Table of Contents
© 2021 College Board
In order for the sphere’s rotation to change direction, a torque must be exerted
on the sphere. When the sphere is at its maximum rightward displacement,
what is the direction of the torque exerted on the sphere with respect to the
point of attachment between the sphere and string?
____ Clockwise ____ Counterclockwise
Briefly state why the torque is in the direction you indicated.
2. A spring with unstretched length L1 is hung vertically, with the top end fixed in
place, as shown in Figure 1 above. A block of mass M is attached to the bottom
of the spring, as shown in Figure 2, and the spring has length L2 > L1 when the
block hangs at rest. The block is then pulled downward and held in place so that
the spring is stretched to a length L3 > L2 , as shown in Figure 3.
(A) On the dot below, which represents the block in Figure 3, draw and label
the forces (not components) exerted on the block. Each force must be
represented by a distinct arrow starting on, and pointing away from, the dot.
AP Physics 1: Algebra-Based Course and Exam Description Exam Information V.1 | 183
Return to Table of Contents
© 2021 College Board
(B) The student releases the block. Consider the time during which the block is
moving upward toward its equilibrium position and the spring length is still
longer than L2.
In a clear, coherent paragraph-length response that may also contain
diagrams and/or equations, indicate why the total mechanical energy is
increasing, decreasing, or constant for each of the systems listed below.
• System 1: The block
• System 2: The block and the spring
• System 3: The block, the spring, and Earth
Use E1, E2, and E3 to denote the total mechanical energy of systems 1, 2, and
3, respectively.
AP Physics 1: Algebra-Based Course and Exam Description Exam Information V.1 | 184
Return to Table of Contents
© 2021 College Board
Answer Key and Question
Alignment to Course Framework
Multiple-Choice
Question Answer Science Practice Learning Objective Unit
1 A 7.2 3.A.4.2 2
2 D 4.2 3.A.1.2 1
3 B 1.4 5.B.4.2 4
4 C 2.2 5.B.3.2 4
5 A 6.4 3.A.3.1 2
6 C 2.2 3.A.1.1 1
7 B 5.1 4.B.2.2 5
8 D 6.4 5.B.5.5 4
9 A 2.2 3.C.1.2 3
10 B 5.1 5.D.2.4 5
11 A 1.4 4.C.1.1 4
12 B, D 7.2 5.E.1.1 7
The scoring information for the questions within this course and exam description, along with further
exam resources, can be found on the AP Physics 1 Exam Page on AP Central.
AP Physics 1: Algebra-Based Course and Exam Description Exam Information V.1 | 185
Return to Table of Contents
© 2021 College Board
THIS PAGE IS INTENTIONALLY LEFT BLANK.
AP PHYSICS 1
Scoring Guidelines
Question 1: Quantitative/Qualitative Translation
1. A small sphere of mass M is suspended by a string of length L. The sphere is made to move in a horizontal circle of
radius R at a constant speed, as shown above. The center of the circle is labeled point C, and the string makes an 0 with
the vertical.
(A) Two students are discussing the motion of the sphere and make the following statements.
Student 1: None of the forces exerted on the sphere are in the direction of point C, the center of the circular path.
Therefore, I don’t see how there can be a centripetal force exerted on the sphere to make it move in a circle.
Student 2: I see another problem. The tension force exerted by the string is at an angle from the vertical. Therefore,
its vertical component must be less than the weight Mg of the sphere. That means the net force on the sphere has a
downward vertical component, and the sphere should move downward as well as moving around in a circle.
i. What is one aspect of Student 1’s reasoning that is incorrect?
ii. What is one aspect of Student 2’s reasoning that is incorrect?
(B)
i. Derive an equation for the magnitude of the net force exerted on the sphere. Express your answer in terms of
M, theta, and physical constants, as appropriate.
ii. Describe one aspect or step in your derivation of part (b)(i) that can be correctly linked to your answer to either
part (a)(i) or part (a)(ii).
AP Physics 1: Algebra-Based Course and Exam Description Scoring Guidelines V.1 | 187
Return to Table of Contents
© 2021 College Board
Instead of moving in a horizontal circle, the sphere now moves in a vertical plane so that it is a simple pendulum,
as shown above. The maximum angle max that the string makes from the vertical can be assumed to be small. The
graph below shows data for the square of the pendulum period T as a function of string length L.
(C) On the graph above, draw a best-fit line for the data. Then use the line to calculate a numerical value for the
gravitational acceleration g .
AP Physics 1: Algebra-Based Course and Exam Description Scoring Guidelines V.1 | 188
Return to Table of Contents
© 2021 College Board
(D) The graph above shows the angle from the vertical as a function of time for the pendulum. On the axes below,
sketch a graph of the gravitational potential energy of the sphere-Earth system for the same time interval. Take the
zero of potential energy to be when the potential energy has its a minimum value.
(E) As the sphere swings back and forth, it must also rotate a small amount during each swing. The figures below
indicate the direction that the sphere rotates as it is swinging in each direction.
In order for the sphere’s rotation to change direction, a torque must be exerted on the sphere. When the sphere is at its
maximum rightward displacement, what is the direction of the torque exerted on the sphere with respect to the point
of attachment between the sphere and string?
____ Clockwise ____ Counterclockwise
Briefly state why the torque is in the direction you indicated.
AP Physics 1: Algebra-Based Course and Exam Description Scoring Guidelines V.1 | 189
Return to Table of Contents
© 2021 College Board
Scoring Guidelines for Question 1: Quantitative/Qualitative
Translation 12 points
(A) i. What is one aspect of Student 1’s reasoning that is incorrect? 1 point
6.4
One point for a claim (explicit or implied) that the sphere can have a centripetal force without any individual
force pointing toward the center of its circular path.
Examples of an acceptable claim:
• The centripetal force is provided by the net force on an object.
• The tension force from the string has a horizontal component, which provides the centripetal force.
ii. What is one aspect of Student 2’s reasoning that is incorrect? 1 point
One point for a claim (explicit or implied) that the tension force is or can be larger than Mg. 6.4
(B) erive an equation for the magnitude of the net force exerted on the sphere, utilizing the terms
i. D 1 point
appropriate terms. 1.4
One point for using Newton’s second law for vertical force components.
∑ Fy = 0
FT cos θ − Mg = 0 or FT cos θ = Mg
One point for writing the horizontal component of string tension in terms of angle. 1 point
1.4
Fnet = Fx = FT sinθ
One point for substituting a correct tension force and writing an answer in terms of the stated quantities. 1 point
2.2
FT cos θ = Mg , so FT = Mg cos θ
Mg
Fnet = FT sinθ = sinθ or Mg tanθ
cos θ
ii. D
escribe one aspect or step in your derivation of part (b)(i) that can be correctly linked to your answer to 1 point
either part (a)(i) or part (a)(ii). 6.4
One point for a correct statement linking the derivation to the answer in either (A)(i) or (A)(ii).
AP Physics 1: Algebra-Based Course and Exam Description Scoring Guidelines V.1 | 190
Return to Table of Contents
© 2021 College Board
(C) Draw a best-fit line for the data. 1 point
One point for drawing a reasonable line of best fit. That is, the straight line drawn should have roughly the 5.1
Then use the line to calculate a numerical value for the gravitational acceleration g. 1 point
One point for correctly calculating the slope or its inverse, using points on the line drawn. 2.2
One point for a calculation of g consistent with the calculated slope or slope inverse. 1 point
2.2
T = 2π L g so T = 4π L g 2 2
Slope = 4π 2 g
(
g = 4π 2 Slope = 4π 2 4.07 s2 m = 9.7 m s2 )
Total for part (C) 3 points
(D) Sketch a graph of the gravitational potential energy of the sphere-Earth system for the same time interval. 1 point
One point for a graph with equal maxima at 0, 0.5 s, and 1 s, and minima of zero at 0.25 s and 0.75 s. 1.5
0
0.0 0.5 1.0
Time (s)
One point for a graph that has even symmetry (mirror symmetry) about time. 1 point
t = 0.5 s 1.5
Notes:
• The maxima should be equal in energy, but may have any energy value.
• A graph with maxima at 0 and 1 s, and a minimum at 0.5 s, can earn the second point only.
AP Physics 1: Algebra-Based Course and Exam Description Scoring Guidelines V.1 | 191
Return to Table of Contents
© 2021 College Board
(E) Briefly state why the torque is in the direction you indicated. 1 point
One point for indicating that the torque is clockwise (claim) with acceptable reasoning that connects the 1.4
claim with evidence. (Note: If the incorrect selection is made, the response is not graded.)
AP Physics 1: Algebra-Based Course and Exam Description Scoring Guidelines V.1 | 192
Return to Table of Contents
© 2021 College Board
Question 2: Paragraph Argument Short Answer
2. A spring with unstretched length L1 is hung vertically, with the top end fixed in place, as shown in Figure 1 above. A
block of mass M is attached to the bottom of the spring, as shown in Figure 2, and the spring has length L2 > L1 when
the block hangs at rest. The block is then pulled downward and held in place so that the spring is stretched to a length
L3 > L2 , as shown in Figure 3.
(A) On the dot below, which represents the block in Figure 3, draw and label the forces (not components) exerted on
the block. Each force must be represented by a distinct arrow starting on, and pointing away from, the dot.
(B) The student releases the block. Consider the time during which the block is moving upward toward its equilibrium
position and the spring length is still longer than L2.
In a clear, coherent paragraph-length response that may also contain diagrams and/or equations, indicate why the
total mechanical energy is increasing, decreasing, or constant for each of the systems listed below.
• System 1: The block
• System 2: The block and the spring
• System 3: The block, the spring, and Earth
Use E1, E2, and E3 to denote the total mechanical energy of systems 1, 2, and 3, respectively.
AP Physics 1: Algebra-Based Course and Exam Description Scoring Guidelines V.1 | 193
Return to Table of Contents
© 2021 College Board
Scoring Guidelines for Question 2: Paragraph Argument Short Answer 7 points
Learning Objectives: 3.B.2.1 3.E.1.2 4.C.1.1 5.A.2.1 5.B.4.1 5.B.4.2
(A) Draw and label the forces (not components) exerted on the block. 1 point
1.1
Accept the following:
FS
FH
FG
One point for including all three labelled forces exerted on the block:
• the upward spring force,
• the downward gravitational force,
• and the downward force of the hand holding the block.
One point for including at least one of the three forces exerted on the block, with no extraneous forces. 1 point
1.1
(B) In a clear, coherent paragraph-length response that may also contain diagrams and/or equations, 1 point
indicate why the total mechanical energy is increasing, decreasing, or constant for each of the systems 6.4
listed below.
• System 1: The block
• System 2: The block and the spring
• System 3: The block, the spring, and Earth
One point for a response with no incorrect claims about which forms of energy are present in each system.
• Note: Responses that do not explicitly refer to the forms of energy in one or more of the systems can
still earn this point.
One point for correctly providing an indication that total mechanical energy is increasing for the block. 1 point
1.4
Examples of acceptable statements:
• The block is accelerating upward with increasing speed.
• The net force on the block is upward, in the direction of the block’s velocity, so the block’s kinetic
energy is increasing.
AP Physics 1: Algebra-Based Course and Exam Description Scoring Guidelines V.1 | 194
Return to Table of Contents
© 2021 College Board
One point for correct statement of why the total mechanical energy is decreasing for the 1 point
block-spring system. 2.1
E 3 is the total mechanical energy of System 3. E 3 is constant, while Ugrav is increasing because the
One point for correct statement that the total mechanical energy is constant for a closed system such as 1 point
the block-spring-Earth system. 6.4
Note: It is not necessary to state that energy is neither dissipated or added by converting other forms of
energy into mechanical energy, since there is no mention of potential causes for these processes (e.g.,
friction to dissipate mechanical energy, or an explosion to add mechanical energy).
One point for a logical, relevant, and internally consistent argument that addresses the required argument, 1 point
explanation or question asked. 1.4
System 3 can be considered a closed system with no external forces exerted on it, so E 3 is constant.
Note that:
where Kblock is the block’s kinetic energy, Uspring is the spring’s potential energy, and Ugrav is the
gravitational potential energy of the block-Earth system.
E 3 is constant, while Ugrav is increasing because the block is moving upward. So E2 = E 3 − Ugrav must be
decreasing.
AP Physics 1: Algebra-Based Course and Exam Description Scoring Guidelines V.1 | 195
Return to Table of Contents
© 2021 College Board
THIS PAGE IS INTENTIONALLY LEFT BLANK.
AP PHYSICS 1
Appendix
AP PHYSICS 1
Table of
Information:
Equations